MLT capstone

Ace your homework & exams now with Quizwiz!

Which of the following is the etiology for most strokes and heart attacks in the United States? A. Consequences of thrombotic events B. Cancer C. Hemorrhage D. Thrombocytopenia

Consequences of thrombotic events

Production of adrenocortical hormones by adrenal cortex Parathyroid hormone Oxytocin Thyroxine Growth hormone FSH Triiodothyronine Calcitonin ACTH LH Aldosterone TSH ADH

ACTH

Reabsorption of water in distal renal tubules Parathyroid hormone Oxytocin Thyroxine Growth hormone FSH Triiodothyronine Calcitonin ACTH LH Aldosterone TSH ADH

ADH

Which of the following hormones is secreted by the posterior pituitary gland? GH ADH

ADH

Which two liver enzymes are seen elevated in hepatobiliary disease? ALP and GGT AST and ALP GGT and AST

ALP and GGT

Hemoglobin H disease results from: A. Absence of 3 of 4 alpha genes B. Absence of 2 of 4 alpha genes C. Absence of 1 of 4 alpha genes D. Absence of all 4 alpha genes

Absence of 3 of 4 alpha genes

Which of the following is associated with Glanzmann's thrombasthenia? A. Normal bleeding time B. Normal ADP aggregation C. Abnormal ristocetin aggregation D. Absence of clot retraction

Absence of clot retraction

Which of the following is associated with Glanzmann's thrombasthenia? A. Normal bleeding time B. Normal ADP aggregation C. Abnormal ristocetin aggregation D. Absence of clot retraction

Absence of clot retraction

In absorption spectrophotometry

Absorbance is directly proportional to concentration

Which of the following diseases is characterized by primary hyperaldosteronism caused by adrenal adenoma, carcinoma, or hyperplasia?

Conn's disease

Tests of pancreatic exocrine function include:

Measurement of serum amylase and lipase activity

Tests of pancreatic exocrine function include: Measurement of serum insulin and glucagon Measurement of serum amylase and lipase activity

Measurement of serum amylase and lipase activity

Listeria monocytogenes is associated with which conditions? A. Neonate infections B. UTI C. Endocarditis D. Otitis media

Neonate infections

Which condition produces the highest elevation of serum lactate dehydrogenase? Myocardial infarction Pernicious anemia

Pernicious anemia

A patient has 20% eosinophils. Which of the following can probably be eliminated? A. Allergy B. Parasitic infection C. Pertussis D. Chronic granulocytic leukemia

Pertussis

The anticonvulsant used to control tonic-clonic (grand mal) seizures is:

Phenytoin

Regan isoenzyme has the same properties as alkaline phosphatase that originates in the: Skeleton Placenta

Placenta

What substance digests the fibrin clot after healing? A. Plasmin B. activated protein C C. tissue plasminogen activator (TPA) D. alpha-2-antiplasmin

Plasmin

What substance digests the fibrin clot after healing? A. Plasmin B. activated protein C C. tissue plasminogen activator (TPA) D. alpha-2-antiplasmin

Plasmin

A patient's blood gas results are as follows: pH = 7.26; dco2=2.0mmol/L; HCO3-=29mmol/L. These results would be classified as:

Respiratory acidosis

The following laboratory results were obtained: Serum Electrolytes Arterial Blood Sodium 136 mEq/L pH 7.32 Potassium 4.4 mEq/L pCO2 79 mm Hg Chloride 92 mEq/L Bicarbonate 40 mEq/L These results are most compatible with: Respiratory alkalosis Respiratory acidosis Metabolic alkalosis Metabolic acidosis

Respiratory acidosis

If polychromasia is increased in the peripheral smear, the ____________ should be elevated. A. White cell count B. Red cell count C. Reticulocyte count D. Basophil count

Reticulocyte count

Increases in blood ammonia levels would be expected in which of the following conditions: Reye's syndrome Obstructive jaundice

Reye's syndrome

Which of the following is associated with the use of aspirin in young children?

Reye's syndrome

The etiologic agents of many common colds are RNA viruses that grow better at 33 degrees C than at 37 degrees C. These viruses are: A. Adenoviruses B. Orthomyxoviruses C. Paramyxoviruses D. Rhinoviruses

Rhinoviruses

Which of the following conditions is associated with hypophosphatemia?

Rickets

The total WBC count is 22 x 109/L. 25 nRBCs per 100 WBCs are seen on the blood smear. What is the corrected WBC count? A. 17.6 x 109/L B. 20.6 x 109/L C. 12.6 x 109/L D. 16.6 x 109/L Uncorrected WBC count x 100/number of NRBCs per 100 WBCs + 100

17.6 x 109/L

What is the normality of 3.g NaOH in 500mL of solution? (GMW =40, Valence=1)

0.15 N

The tourniquet should be left on no more then ___________ or _____________ will be elevated. 1 min, sodium and bilirubin 5 min, glucose and potassium 1 min, potassium and total protein 45 seconds, potassium and sodium

1 min, potassium and total protein

Calculate the absolute lymphocyte count. RBC 3.0 x 10^12/L WBC 7.0 x 10 ^9/L Neutrophils 56% Lyphocytes 23% Monocytes 14% Eosinophil 6% Basophil 1% A. 1.6 x 10 ^9/L B. 2.3 x 10 ^9/L C. 2.0 x 10 ^12/L D. 1.7 x 10 ^12/L 1.6 x 10 ^9/L

1.6 x 10 ^9/L

What is the SD of the following values: 65,64,68,67,63,64,65 2.53 3.62 1.77 1.02

1.77

Given these values: 100, 120, 150, 140, 130. What is the mean?

128

Calculate LDL cholesterol concentration if total cholesterol is 230 mg/dL, Triglyceride is 140 mg/dL and the HDL cholesterol is 50.0 mg/ dL. Unable to calculate 152 mg/dL

152 mg/dL

What is the percentage of serum calcium that is ionized?

45%

Given the information below, determine the concentration of the unknown. Patient Absorbance: 0.740 Standard Absorbance: 0.860 Concentration of Standard: 200mg/dL A. 123 mg/dL B. 172 mg/dL C. 232 mg/dL D. 314 mg/dL

172 mg/dL

Given the information below, determine the concentration of the unknown. Patient Absorbance: 0.740 Standard Absorbance: 0.860 Concentration of Standard: 200mg/dL 182 mg/dL 172 mg/dL 165 mg/dL 162 mg/dL

172 mg/dL

A tube contains the following: Serum 0.25 mL, Saline 0.75mL. What is the dilution?

1:4

How soon following acute abdominal pain due to pancreatitis is the serum amylase level expected to rise?

2-12 hours

Most patients on warfarin therapy should maintain an international normalized ratio (INR) of approximately ____ times of a normal value. A. 0.5-1.5 B. 1-2 C. 2-3 D. 3-4

2-3

Absorbance (A) of a solution may be converted to percent transmittance (%T) using the formula: 2-log%T 2+log%T

2-log%T

Convert 10 mg/dL of calcium to mmole/L. (mw = 40)

2.5

Convert 10 mg/dL of calcium to mmole/L. (mw = 40) 2.5 5 10 25

2.5

Which of the following values is the threshold critical value for low plasma potassium? 2.5mmol/L 2.0mmol/L

2.5mmol/L

How many milliliters of 0.5 N NaOH are required to make 50 milliliters of 0.2N NaOH?

20

Calculate the MCH. RBC 3.0 x 10^12/L Hgb 6g/dL HCT 20% A. 30 g/dL B. 32 g/dL C. 20 fg D. 25 fg

20 fg

Which of the following would be an unexpected finding in chronic myeloid leukemia (CML)? A. increased eosinophils and basophils in the peripheral blood B. 20% blasts in the peripheral blood C. hypercellular bone marrow with granulopoiesis D. white blood count of 100 x 109/L

20% blasts in the peripheral blood

A patient sample for uric acid gave an absorbance reading of 0.4; the 50 mg/dL standard used in the test gave an absorbance reading of 0.1. What is the concentration of uric acid in the patient's sample in grams per deciliter? 0.2 12.5 125 200

200

What is the normal ratio of bicarbonate to dissolved carbon dioxide in arterial blood?

20:1

What is the MCH if the hematocrit is 20%, the RBC is 2.4 x 106/µL, and the hemoglobin is 5 g/dL? A. 21 pg B. 23 pg C. 25 pg D. 84 pg

21 pg

A 1:100 dilution is made and the 4 corner squares are counted. 95 WBCs are counted on one side and 89 on the other side. Calculate the WBC count. A. 33 x 10^9/L B. 23 x 10^9/L C. 20 x 10^9/L D. 13 x 10^9/L

23 x 10^9/L

Given the following data: WBC 8.5 x 103/µL Differential Segs 56% Bands 2% Lymphs 30% Monos 6% Eos 6% What is the absolute lymphocyte count? A. 170 B. 510 C. 2550 D. 4760

2550

A patient has a Hgb of 9 g/dL. Using the rule of three, what would you expect the Hematocrit to be? A. 23 ± 3 B. 33 ± 3 C. 27 ± 3 D. 45 ± 3

27 ± 3

patient has a Hgb of 9 g/dL. Using the rule of three, what would you expect the Hematocrit to be? A. 23 ± 3 B. 33 ± 3 C. 27 ± 3 D. 45 ± 3

27 ± 3

A patient has the following serum results: Na = 134 mEq/L K = 4.5 mEq/L Cl = 108 mEq/L Glucose = 250 mg/dL BUN = 15 mg/dL Creatinine = 0.8 mg/dL What is the osmolality result? 178 203 165 287

287

The expected life span of a normal red blood cell is: A. 3-4 months B. 1-2 months C. 3-6 weeks D. 1-3 weeks

3-4 months

If a patient has a reticulocyte count of 7% and an Hct of 20%, what is the corrected reticulocyte count? A. 2.1% B. 3.1% C. 3.5 % D. 4.3%

3.1%

The appropriate anticoagulant for prothrombin time assay is: A. EDTA B. 3.2% sodium citrate C. 3.8% sodium citrate D. Heparin

3.2% sodium citrate

A patient sample gave an absorbance reading of 0.2; the 5 mg/dL standard used in the test gave an absorbance reading of 0.3. What is the concentration in the patient's sample? 3.3 4.3 4.5 3.8

3.3

A 1:200 dilution of a patient's sample was made and 336 red cells were counted in an area of 0.2 mm2. What is the RBC count? A. 1.68 x 10^12/L B. 3.36 x 10^12/L C. 4.47 x 10^12/L D. 6.66 x 10^12/L

3.36 x 10^12/L

Calculate the MCHC. RBC 3.0 x 10^12/L Hgb 6g/dL HCT 20% A. 30 g/dL B. 32 g/dL C. 20 pg D. 25 pg

30 g/dL

From where are additional platelets first derived when the circulating count decreases? A. cells marginated on vessel endothelial cells B. 30% of mature cells that are stored in the spleen C. bone marrow D. stem cells

30% of mature cells that are stored in the spleen

Serum glucose determinations are done on samples from a control pool each day for 30 days. Quality control statistics are then calculated. The mean is 100 mg/dL and the standard deviation is 4 mg/dL. These statistics are used to label a quality control chart for the mean and +2 SD. Samples from the same control pool are then assayed each day along with patient samples, and control results are plotted on the chart. Results for the first 10 days are as follows:Day Result1 962 973 954 1075 1066 1077 898 1039 10610 105The coefficient of variation for these statistics is: 4% 8% 12.5% 25%

4%

Anaerobes grow best in: Answers: A. ambient air, which contains 21% oxygen (O2) and a small amount (0.03%) of carbon dioxide B. increased concentrations of CO2 (5% to 10%) and approximately 15% O2 C. 5% to 10% hydrogen (H2), 5% to 10% CO2, 80% to 90% nitrogen (N2), and 0% O2 D. reduced O2 (5% to 10%) and increased CO2 (8% to 10%)

5% to 10% hydrogen (H2), 5% to 10% CO2, 80% to 90% nitrogen (N2), and 0% O2

What percentage of dietary iron is normally absorbed daily? 15% to 20% 3% to 10% 5% to 15% 10% to 15%

5% to 15%

Paired, acute, and convalescent blood samples are needed for diagnostic viral serology testing. Which of the following show the recommended intervals (Acute/Convalescent) for collection? A. 13-15 days / 50-60 days B. 11-13 days / 40-50 days C. 7-9 days / 20-30 days D. 5-7 days / 10-20 days

5-7 days / 10-20 days

Calculate the creatinine given the following results: Urine Creatinine: 98mg/dl 24 hour Urine Volume:2550 mL Plasma Creatinine: 1.1 mg/dL BSA: 1.63 m2

54.99

Calculate the MCV. RBC 3.0 x 10^12/L Hgb 6g/dL HCT 20% A. 67 fl B. 20 pg C. 30 g/dL D. 73 fl

67 fl

How many grams of CaSO4 are needed to prepare one liter of a 0.5M solution? (GMW= 136) 27.2 g 65 g

68 g

Which of the following best represents the reference (normal) range for arterial pH?

7.35-7.45

Calculate the coefficient of variation for a set of data where the mean = 89 mg/dL and 2 standard deviations is 14. 7.6% 7.7% 7.4% 7.9%

7.9%

Calculate the creatinine given the following results: Urine Creatinine: 121mg/dL 24 hour Urine Volume: 1950 mL Plasma Creatinine: 1.8 mg/dL BSA: 1.76 m 2 90 ml/mon 32 ml/min 89.2 ml/min

89.2 ml/min

In quality control, ±2 standard deviations from the mean includes what percent of the sample population? 95 75 80 90

95

Oliguria is: <100 ml of urine output daily <400 ml of urine output daily

<400 ml of urine output daily

Adult (normal) Hemoglobin is made up of the following composition: A. >90% HbA, 5%HbF, 1% HbA2 B. >95% HbA, <3.5% HbA2, <1-2% HbF C. <90% HbA, 10% HbA2, 5% HbF D. >90% HbA, 1% HbA2, 5-10% HbF

>95% HbA, <3.5% HbA2, <1-2% HbF

The final result of secondary hemostasis is: A. Platelet plug B. Constriction of a blood vessel C. A blood clot D. Promotion of bleeding to cleanse a wound

A blood clot

What is the basic hemoglobin defect in the thalassemias? A. One of the globin chains has an amino acid substitution B. A structurally normal globin chain is absent or produced at lower levels C. Heme is produced at a lower concentration D. Iron is not incorporated into the protoporphyrin ring to form heme

A structurally normal globin chain is absent or produced at lower levels

In absorption spectrophotometry: Absorbance is directly proportional to transmittance Percent transmittance is directly proportional to concentration Percent transmittance is directly proportional to the light path length Absorbance is directly proportional to concentration

Absorbance is directly proportional to concentration

In a spectrophotometric determination, which one of the following is the formula for calculating the absorbance of a solution? (Absorptivity x concentration) / light path in cm (Absorptivity x light path in cm) / concentration Absorptivity x light path in cm x concentration (Light path in cm x concentration) / absorptivity

Absorptivity x light path in cm x concentration

What is the metabolite of ethanol?

Acetaldehyde

Mycobacteria spp. can be detected by all of the following methods except A. Ziehl-Neelson B. Auramine-rhodamine C. Acridine orange D. Kinyoun

Acridine orange

Children who have infections with beta-hemolytic streptococci can develop: A. Acute Pyelonephritis B. Chronic glomerulonephritis C. Acute glomerulonephritis D. Nephrosis

Acute glomerulonephritis

Given the following results: ALP Slight increase ALT Marked increase AST Marked increase GGT Slight increase This is most consistent with Obstructive jaundice Acute hepatitis

Acute hepatitis

Defect in enzyme tyrosine Maple syrup urine disease Alkaptonuria Tyrosinemia Phenylketonuria Albinism Cystinuria

Albinism

In the liver, methanol is metabolized by ________ to form __________. Alcohol dehydrogenase; acetaldehyde Alcohol dehydrogenase; formaldehyde

Alcohol dehydrogenase; formaldehyde

Reabsorption of sodium in the renal tubules Parathyroid hormone Oxytocin Thyroxine Growth hormone FSH Triiodothyronine Calcitonin ACTH LH Aldosterone TSH ADH

Aldosterone

The adrenal cortical hormone that functions to promote sodium resorption by the renal tubules and thus affect water balance in the body is:

Aldosterone

Which hormone controls sodium reabsorption?

Aldosterone

Which of the following is the major mineralocorticoid?

Aldosterone

Which of the following is the major mineralocorticoid? Aldosterone Cortisol Corticosterone Testosterone

Aldosterone

Which of the following conditions is associated with hypokalemia? Addison's disease Alkalosis

Alkalosis

Match the disease with the cause: Deficiency in homogentisate oxidase Maple syrup urine disease Alkaptonuria Tyrosinemia Phenylketonuria Albinism Cystinuria

Alkaptonuria

In an adult, hematopoiesis occurs in the marrow of which of the following bones? A. Vertebrae B. Skull C. Proximal ends of long bones D. All answers are correct

All answers are correct

The type of vaginitis referred to as bacterial vaginosis is thought to be caused by: A. Gardnerella vaginalis. B. Prevotella spp. C. peptostreptococci. D. Mobiluncus spp. E. All of these are correct.

All of these are correct.

The following polypeptide chains are found in normal adult hemoglobin A: A. Alpha and gamma B. Alpha and delta C. Beta and delta D. Alpha and beta

Alpha and beta

The acute phase reactant protein that also inhibits proteolysis and has the highest concentration within its globulin fraction is: C-reactive protein Haptoglobin Alpha2-macroglobulin Alpha1-antichymotrypsin Alpha1-antitrypsin

Alpha1-antitrypsin

A physician suspects his patient has pancreatitis. Which test(s) would be most indicative of this disease? Amylase ID Isoenzymes

Amylase

Which one of the following statements concerning cortisol is not correct? Plasma cortisol levels show a diurnal variation with the highest level present in the morning and the lowest level in the late afternoon. An ectopic cortisol-secreting tumor can lead to spontaneous Cushing syndrome.

An ectopic cortisol-secreting tumor can lead to spontaneous Cushing syndrome.

All of the following are considered routine coagulation tests that are available in most large and small clinical laboratories except: A. Antithrombin assay B. Prothrombin time C. Fibrinogen assay D. Partial thromboplastin time

Antithrombin assay

Which of the following Haemophilus spp. not require neither X or V factors for growth? A. Influenzae B. Parainfluenzae C. Aphrophilus D. Ducreyi

Aphrophilus

Which apoprotein is inversely related to risk for coronary heart disease?

Apoprotein A-I

The glucose value of a NORMAL 2-hour post-prandial serum specimen, as compared to the reference range for a fasting serum glucose, should be: Significantly greater than the normal fasting glucose level Significantly lower than the normal fasting glucose level Approximately the same as the normal fasting glucose level Unrelated to each other

Approximately the same as the normal fasting glucose level

This fungal species shown below was isolated from the blood of a bone marrow transplant patient. What is the most likely identification? A. Aspergillus B. Penicillium C. Rhizopus species D. Mucor species

Aspergillus

Which of the following vitamins is NOT a fat-soluble vitamin?

B1

Which of the following vitamins is NOT a fat-soluble vitamin? D B1

B1

A patient with glomerulonephritis would present the following serum results: BUN increased Creatinine decreased

BUN increased

Fluid from a cutaneous black lesion was submitted for routine bacteriological culture. After 18 hours of incubation at 35C there was no growth on MacConkey agar, but 3+ growth on sheep blood agar. The colonies were nonhemolytic, 4-5 mm in diameter, and off-white with a ground glass appearance. Each colony had an irregular edge with comma-shaped outgrowths that stood up like "beaten egg whites" when gently lifted with an inoculating loop. A gram-stained smear of a typical colony showed large, gram positive, rectangular rods. The organism is most likely: A. Clostridium perfringens B. Aeromonas hydrophilia C. Bacillus anthracis D. Corynebacterium diphtheriae

Bacillus anthracis

Suspect spore-forming gram-positive bacilli that are nonhemolytic, nonmotile, and penicillin-susceptible, and that produce a wide zone of lecithinase on egg yolk agar can be identified as: A. Corynebacterium diphtheriae B. Bacillus anthracis C. Bacillus cereus D. Corynebacterium jeikeium

Bacillus anthracis

Suspect spore-forming gram-positive bacilli that are hemolytic, motile, and penicillin-resistant and that produce a wide zone of lecithinase on egg yolk agar can be identified as: A. Corynebacterium diphtheriae B. Bacillus anthracis C. Bacillus cereus D. Corynebacterium jeikeium

Bacillus cereus

Why is the absolute neutrophil count (ANC) monitored closely in patients receiving chemotherapy? A. Viral infection is a possibility when ANC is <0.5 x 109/L B. Severe anemia will develop when the ANC is <1.0 x 109/L C. Bacterial infection becomes a possibility when the ANC is <1.0 x 109/L D. Platelet count will decrease when the ANC is <0.5 x 109/L

Bacterial infection becomes a possibility when the ANC is <1.0 x 109/L

The primary buffer of plasma is:

Bicarbonate

Eikenella corrodens A. Bleach odor B. Strawberry odor C. Dirt like odor D. Grape odor E. Mouse nest odor

Bleach odor

Pertussis (whooping cough), an epidemic disease, is a highly contagious, acute infection of the upper respiratory tract caused by: A. Brucella species B. Bordetella species C. Pasteurella species D. Francisella species

Bordetella species

An unbalanced hemostatic mechanism can produce: A. Thrombosis B. Bleeding C. Inadequate number of platelets D. Both thrombosis and bleeding

Both thrombosis and bleeding

Bacillus: A. Boxcar shape B. Cat eyes C. Chinese letters D. Cluster of grapes E. Branching F. Safety pins G. String of pearls

Boxcar shape

Nocardia: A. Boxcar shape B. Cat eyes C. Chinese letters D. Cluster of grapes E. Branching F. Safety pins G. String of pearls

Branching

Which of the following bacteria is a gram negative rod with a dirt-like odor? A. Alcaligenes faecalis B. Pseudomonas aeruginosa C. Burkholderia cepacia D. Stenotrophomonas maltophilia

Burkholderia cepacia

Which of the following bacteria is associated with glanders disease? A. Burkholderia cepacia B. Burkholderia pseudomallei C. Ralstonia pickettii D. Burkholderia mallei

Burkholderia mallei

Hypernatremia commonly occurs in: Burns, syndrome of inappropriate ADH, and excessive sweating without water intake Burns and excessive sweating without water intake Syndrome of inappropriate ADH and nephrotic syndrome Nephrotic syndrome only Burns, syndrome of inappropriate ADH, excessive sweating without water intake, and nephrotic syndrome

Burns and excessive sweating without water intake

A deficiency of which vitamin will produce abnormal bone mineralization and eventually osteomalacia A E D C

C

Scurvy is associated with deficiency of which of the following vitamins?

C

Scurvy is associated with deficiency of:

C

Which of the following causes periodontal disease? A. C. coli B. C. curvus C. C. jejuni D. Arcobacter butzleri

C. curvus

Which of the following Campylobacter spp. can be separated from the other species by a positive hippurate hydrolysis test? A. C. coli B. C. curvus C. C. concisus D. C. jejuni

C. jejuni

Which of the Which of the following is the best analyte to monitor for recurrence of ovarian cancer?

CA-125

Which of the following is the proper designation for the pluripotential stem cell that is a precursor for both granulocytes and erythrocytes cell lines: A. CFU-S B. CFU-GEMM C. G-CSF D. CFU-GM

CFU-S

When myocardial infarction occurs, the first enzyme to become elevated is:

CK

Inhibition of calcium reabsorption Parathyroid hormone Oxytocin Thyroxine Growth hormone FSH Triiodothyronine Calcitonin ACTH LH Aldosterone TSH ADH

Calcitonin

A hospitalized patient is experiencing increased neuromuscular irritability (tetany). Which of the following tests should be ordered immediately

Calcium

The prothrombin time test requires that the patient's plasma be combined with: A. Platelet lipids B. Thromboplastin C. Calcium and activator D. Calcium and thromboplastin

Calcium and thromboplastin

Which of the following stains greatly enhances the visibility of fungi by binding to the cell walls, causing the fungi to fluoresce blue-white or apple green? A. Rhodamine-auramine B. Warthin-Starry C. Calcofluor white D. Periodic Acid-Schiff

Calcofluor white

An international normalized ratio (INR) of 6.5 is obtained on a patient taking warfarin. All quality control is acceptable. What should be done? A. Report the result B. Call the health care provider immediately C. Report only the prothrombin time in seconds and ignore the INR D. Send an e-mail to the health care provider

Call the health care provider immediately

An international normalized ratio (INR) of 6.5 is obtained on a patient taking warfarin. All quality control is acceptable. What should be done? A. Report the result B. Call the health care provider immediately C. Report only the prothrombin time in seconds and ignore the INR D. Send an e-mail to the health care provider

Call the health care provider immediately

Stool received for routine culture in most clinical laboratories in the United States should be examined for the presence of at least: A. Campylobacter, Salmonella, and Shigella spp. B. Salmonella and Shigella spp. C. Campylobacter and Vibrio D. Salmonella, Shigella, and Vibrio

Campylobacter, Salmonella, and Shigella spp.

Stool received for routine culture in most clinical laboratories in the United States should be examined for the presence of at least: A. Campylobacter, Salmonella, and Shigella spp. B. Salmonella and Shigella spp. C. Campylobacter and Vibrio. D. Salmonella, Shigella, and Vibrio.

Campylobacter, Salmonella, and Shigella spp.

Random-access analysis is a type of analysis in which a specimen: Can be analyzed only in continuous sequence with other specimens Is subjected to a single process that produces results for a single analyte Can be analyzed by any available process without regard to the initial order of specimens Can be analyzed by any available process but only in the initial order of specimens

Can be analyzed by any available process without regard to the initial order of specimens

A potentially pathogenic yeast that is normal flora in the oropharyngeal cavity and may produce thrush is: A. Trichosporon beigelii B. Candida albicans C. Cryptococcus neoformans D. Geotrichum

Candida albicans

The formation of germ tubes presumptively identifies: A. Candida tropicalis B. Candida parapsilosis C. Candida glabrata D. Candida albicans

Candida albicans

Increased concentrations of CO2 (5% to 10%) and approximately 15% O2 is the environmental condition that best suits which type of organism? A. Aerobes B. Anaerobes C. Capnophiles D. Microaerophiles

Capnophiles

Streptococcus pneumoniae: A. Boxcar shape B. Cat eyes C. Chinese letters D. Cluster of grapes E. Branching F. Safety pins G. String of pearls

Cat eyes

Hemolysis in paroxysmal nocturnal hemoglobinuria (PNH) is: A. Temperature dependent B. Complement independent C. Antibody mediated D. Caused by a red cell membrane defect

Caused by a red cell membrane defect

Corynebacterium diphtheriae: A. Boxcar shape B. Cat eyes C. Chinese letters D. Cluster of grapes E. Branching F. Safety pins G. String of pearls

Chinese letters

Clonorchis sinensis: A. "Old World" cutaneous leishmaniasis B. Visceral leishmaniasis C. Beef tape worm D. River blindness E. Passed as larvae F. Whipworm G. Hookworm H. Ciliate I. Chagas disease J. Elephantiasis K. Chinese liver fluke L. Diagnosed in sputum sample M. African eye worm N. Amebic dysentery O. Found in muscle P. Waterborne outbreaks

Chinese liver fluke

The anion with the highest extracellular concentration is: Chloride Sodium

Chloride

Of the following media, which provides the factors necessary for the growth of Haemophilus spp.? A. 5% sheep blood agar B. Brain heart infusion agar C. Chocolate agar D. Nutrient agar E. All of these are correct.

Chocolate agar

The parent substance in the biosynthesis of androgens and estrogens is: Cortisol Catecholamines Progesterone Cholesterol

Cholesterol

A thin creamy layer is found on a stored plasma tube, this suggests the presence of:

Chylomicrons

The most likely cause for serum/plasma to appear "milky" is the presence of:

Chylomicrons

Why is it important to monitor patients who are receiving anticoagulant therapy? A. Clinical consequences for overdosing or underdosing are significant B. Prophylactic and therapeutic dosage ranges are unknown C. Patients commonly abuse these drugs D. Drugs are expensive

Clinical consequences for overdosing or underdosing are significant

An anaerobic, spore-forming, gram positive bacillus isolated from a deep wound of the leg is most probably: A. Francisella tularensis B. Clostridium perfringens C. Bacillus subtilis D. Bacteroides

Clostridium perfringens

Staphylococcus: A. Boxcar shape B. Cat eyes C. Chinese letters D. Cluster of grapes E. Branching F. Safety pins G. String of pearls

Cluster of grapes

The most critical distinction between Staphylococcus aureus and other staphylococci is A. Phosphatase reaction B. DNA production C. Coagulase production D. Hemolysis

Coagulase production

Thayer-Martin has 4 antibiotics, match up what each antibiotic inhibits. What inhibits gram negative? A. Colistin B. Trimethoprim C. Nystatin D. Vancomycin

Colistin

The persistent survival of microorganisms on a surface of the human body is called: A. Infection B. Colonization C. Nosocomial infection D. Reservoir

Colonization

A gram stain from a swab of a hand wound reveals: Moderate gram positive cocci in chains Moderate large gram negative bacilli Select the appropriate media that will selectively isolate each organism. A. KV-laked agar, Thayer-Martin B. Sheep blood, MacConkey C. Columbia CNA, Chocolate D. Columbia CNA, MacConkey

Columbia CNA, MacConkey

A clinical laboratory receives a new lot of partial thromboplastin time (PTT) reagent, so clinical laboratory scientists in the laboratory need to establish the heparin therapeutic range for this new reagent lot. How should this be done? A. Add heparin at various therapeutic concentrations to normal plasma, and perform PTT on each concentration using the new lot of reagent B. Add heparin at high concentration to one normal plasma, make dilutions of this plasma, and then perform PTT on each diluted sample using the new lot of reagent C. Perform chromogenic Xa and PTT assays on patient heparinized samples, and do a statistical analysis of result comparisons D. Compare PTT results for patient heparinized samples to those for the lot of PTT reagent that is presently being used

Compare PTT results for patient heparinized samples to those for the lot of PTT reagent that is presently being used

Factors XI, XII, prekallikrein, and HMWK tissue group Contact group prothrombin group

Contact group

An IU of enzyme activity is the quantity of enzyme that: Converts 1 mmol of Substrate per liter Converts 1 mmol of substrate to product per minute

Converts 1 mmol of substrate to product per minute

Pituitary secretion of adrenocorticotropic hormone (ACTH) is inhibited by elevated levels of:

Cortisol

Which of the following is known to cause Q fever? A. Bartonella henselae B. Bartonella quintana C. Coxiella D. Rickettsia rickettsii

Coxiella

The highest levels of this enzyme are seen in muscular dystrophy.

Creatine Kinase

Rickets is associated with deficiency of which of the following vitamins? A B C D

D

A patient develops unexpected bleeding and the following test results were obtained: Prolonged PT and APTT Decreased fibrinogen Increased fibrin split products Decreased platelets What is the most probable cause of these results? A. Familial afibrinogenemia B. Primary fibrinolysis C. DIC D. Liver disease

DIC

A patient develops unexpected bleeding and the following test results were obtained: Prolonged PT and APTT Decreased fibrinogen Increased fibrin split products Decreased platelets What is the most probable cause of these results? A. Familial afibrinogenemia B. Primary fibrinolysis C. DIC D. Liver disease

DIC

Howell-Jolly bodies are composed of: A. Hemoglobin B. DNA C. Iron D. Phospholipids

DNA

Megaloblastic anemias are caused by a defect in the synthesis of: A. DNA B. RNA C. Erythropoietin D. Heme

DNA

Which one of the following sets of laboratory results is consistent with hemolytic anemia? A. Increased concentration of haptoglobin; negative hemoglobinuria B. Decreased erythrocyte survival; increased catabolism of heme; decreased haptoglobin levels C. Decreased serum LDH activity; normal catabolism of heme D. Normal concentration of haptoglobin; marked hemoglobinuria

Decreased erythrocyte survival; increased catabolism of heme; decreased haptoglobin levels

Lab findings in hereditary spherocytosis do NOT include: A. Reticulocytosis B. Shortened erythrocyte survival C. Decreased osmotic fragility D. Increased MCHC

Decreased osmotic fragility

Which of the following platelet responses is most likely associated with Bernard Soulier Syndrome? A. Decreased platelet aggregation to ristocetin B. Defective ADP release, normal response to ADP C. Decreased amount of ADP in platelets D. Markedly decreased aggregation to epinephrine, ADP, and collagen

Decreased platelet aggregation to ristocetin

Which of the following is likely to occur first in iron deficiency anemia?

Decreased serum ferritin

What is the primary function of leukocytes as a whole? A. Defend body from nonself agents B. Kill parasites C. Make antibodies D. Engulf bacteria

Defend body from nonself agents

Increased serum albumin concentrations are seen in which of the following conditions?

Dehydration

In a suspected case of Hansen's disease, a presumptive diagnosis is established by: A. Isolation of organisms on Lowenstein-Jensen medium B. Detection of weakly acid-fast rods in infected tissue C. Isolation of organisms on Loeffler's medium D. Detection of gram positive rods in infected tissue

Detection of weakly acid-fast rods in infected tissue

Which of the following conditions is associated with hypernatremia?

Diabetes insipidus

Porphobilinogen may be differentiated from urobilinogen in urine by: Difference in chloroform solubility of Ehrlich's aldehyde derivatives Difference in reactivity with Ehrlich's aldehyde reagent Difference in fluorescence of Ehrlich's aldehyde derivatives Difference in precipitation with ammonium sulfate

Difference in chloroform solubility of Ehrlich's aldehyde derivatives

A cardiac glycoside that is used in the treatment of congenital heart failure and arrhythmias by increasing the force and velocity of myocardial contraction is:

Digoxin

Burkholderia cepacia A. Bleach odor B. Strawberry odor C. Dirt like odor D. Grape odor E. Mouse nest odor

Dirt like odor

Which of the following best describes the correct collection and handling conditions for a blood ammonia measurement?

Draw heparin tube, keep on ice, test immediately

Most standards organizations recognize _______ as the anticoagulant of choice for blood cell counts. Sodium citrate Sodium fluoride EDTA Heparin

EDTA

All of the following are true concerning the Erythrocyte Sedimentation Rate EXCEPT: A. Can be used to follow the course of a disease B. ESR is decreased during inflammatory conditions C. Highest ESR values usually seen in Multiple Myeloma D. Increased in rheumatoid conditions

ESR is decreased during inflammatory conditions

The bacterial species that can be described as able to grow in 6.5% NaCl and 40% bile, part of the normal fecal flora of animals and humans, usually nonhemolytic, and a major cause of nosocomial infections is: A. Enterococcus faecalis B. Streptococcus pneumoniae C. Streptococcus pyogenes D. Streptococcus agalactiae

Enterococcus faecalis

Which toxin produced by Streptococcus pyogenes will produce the characteristic rash seen in scarlet fever? A. Erythrogenic toxin B. Hyaluronidase C. M protein D. Streptolysin S

Erythrogenic toxin

The most common etiologic agent in uncomplicated community-acquired urinary tract infection is: A. Escherichia coli B. Klebsiella spp C. Staphylococcus saprophyticus D. Streptococcus pyogenes

Escherichia coli

Which of the following organisms can grow in the small bowel and cause diarrhea in children, traveler's diarrhea, or severe cholera-like syndrome through the production of enterotoxins? A. Yersinia enterocolitica B. Escherichia coli C. Salmonella typhi D. Shigella dysenteriae

Escherichia coli

Select the main estrogen that is produced by the ovaries and is used to evaluate ovarian function. Estradiol hydroxyestrone

Estradiol

When a purified enzyme is used as a reagent, such as urease in the measurement of urea, the enzyme must be ___________________ concentration so that the reaction follows_________________ order kinetics. Excess; first Excess; zero

Excess; first

The pathway taken when tissue thromboplastin, a substance not found in the blood, enters the vascular system and, in the presence of calcium and factor VII, activates factor X Extrinsic pathway intrinsic pathway

Extrinsic pathway

Which of the following factors is Fibrinogen? A. Factor II B. Factor I C. Factor III D. Factor V

Factor I

Which of these factors may be deficient if both the PT and aPTT are prolonged? A. Factor II B. Factor VII C. Factor VIII D. Factor XI

Factor II

The thrombin time will be prolonged in the presence of all the following except: A. Elevated fibrinogen degradation products B. Heparin C. Factor I deficiency D. Factor II deficiency

Factor II deficiency

Which of the following factors is Tissue factor? A. Factor IV B. HMWK C. Factor III D. Factor XII

Factor III

what is the ionized calcium ( Ca++ ) factor? A. Factor IV B. Factor XI C. Factor V D. Factor III

Factor IV

Which of the following factors is Christmas factor? A. Factor VIII B. Factor IV C. Factor V D. Factor IX

Factor IX

Which of the following factors is antihemophilic factor? A. Factor XI B. Factor IX C. Factor V D. Factor VIII

Factor VIII

A 53-year-old woman presents to the hospital with bleeding into the back of the throat and has a coagulation screen performed. She has no history of abnormal bleeding prior to this despite major stress to hemostasis in the past: she was in a major automobile accident at age 37 and has had impacted wisdom teeth extracted. Her coagulation results follow: PT: 13.1 seconds range 11-15 seconds PTT: 87 seconds range 25-37 seconds Fibrinogen: 380 mg/dL range 200-400 mg/dL PTT with 1:1 normal plasma: 35 seconds Which of the following is the most likely etiology for her bleeding? A. Factor XII deficiency B. Factor VIII inhibitor C. Factor VIII deficiency D. Factor IX deficiency

Factor VIII deficiency

A 37-year-old female patient has a partial thromboplastin time (PTT) of 76 seconds (reference range 25 to 37 seconds). Her plasma is mixed 1:1 with normal plasma. An immediate PTT performed on the mix is 74 seconds. Which of the following is the most likely diagnosis? A. Hemophilia A B. Intrinsic factor deficiency C. Factor VIII inhibitor D. Lupus anticoagulant (LA)

Factor VIII inhibitor

Which of the following is NOT produced by the islets of Langerhans of the pancreas? Gastrin Glycogen

Glycogen

The physiological catabolism of glucose to pyruvate for ATP production is referred to as:

Glycolysis

How does hemophilia A differ from hemophilia B? A. Hemophilia A is an inherited disorder and hemophilia B is an acquired disorder. B. The symptoms are drastically different. C. Factor VIII is deficient in hemophilia A and factor IX is deficient in hemophilia B.

Factor VIII is deficient in hemophilia A and factor IX is deficient in hemophilia B

Which of the following factors is Stuart-Prower factor? A. Factor X B. Factor XI C. Factor I D. Prekallikrein

Factor X

Lymphocytes typically contain many specific cytoplasmic granules. A. True B. False

False

Macrophages are actually lymphocytes that have entered the tissues and body fluids via diapedesis. A. True B. False

False

True or false? Peak drug levels in the blood are usually measured to determine the drug's efficacy. True False

False

True or false? The main function of antidiuretic hormone is to increase the reabsorption of sodium and increase the secretion of potassium. True False

False

Organisms that have nutritional needs that are relatively complex and require extra media components to be used for growth are called: A. Fastidious B. Capnophiles C. Neutrophils D. Halophiles

Fastidious

Organisms that have nutritional needs that are relatively complex and require extra media components to be used for growth are called: A. Halophiles B. Capnophiles C. Microaerophiles D. Fastidious

Fastidious

The final end-product of fibrinolysis is: A. Fibrinogen B. Fibrin C. Fibrin-split products D. Plasmin

Fibrin-split products

Which of the following is decreased in disseminated intravascular coagulation (DIC)? A. Fibrinogen B. PT C. PTT D. FDP

Fibrinogen

Which of the following is decreased in disseminated intravascular coagulation (DIC)? A. Fibrinogen B. PT C. PTT D. FDP

Fibrinogen

Which of the following forms of calcium is biologically active:

Free ionized calcium

Select the most sensitive marker for alcoholic liver disease.

GGT

An example of a hormone synthesized in the adenohypophysis would be: GH Aldosterone

GH

What platelet membrane receptor binds fibrinogen and supports platelet aggregation? A. GP Ib/IX/V B. GP IIb/IIIa C. GP Ia/IIa D. P2Y1

GP IIb/IIIa

What platelet membrane receptor binds fibrinogen and supports platelet aggregation? A. GP Ib/IX/V B. GP IIb/IIIa C. GP Ia/IIa D. P2Y1

GP IIb/IIIa

Of the following protein fractions, the one that is NOT produced in the liver is: Albumin Gamma globulin

Gamma globulin

Which of the following bacteria can be seen in epithelial cells called "clue cells"? A. Lactobacillus B. Corynebacterium diphtheriae C. Streptococcus agalactiae D. Gardnerella vaginalis

Gardnerella vaginalis

Which of the following antimicrobial agents act by inhibiting protein synthesis? A. Rifampin B. Methicillin C. Gentamicin D. Vancomycin

Gentamicin

Which of the following is characteristic of Bernard-Soulier Syndrome? A. Giant platelets B. Normal bleeding time C. Abnormal aggregation with ADP D. Increased platelet count

Giant platelets

Which of the following is characteristic of Bernard-Soulier Syndrome? A. Giant platelets B. Normal bleeding time C. Abnormal aggregation with ADP D. Increased platelet count

Giant platelets

Creatinine clearance is used to estimate the

Glomerular filtration rate

Which enzyme is responsible for the conjugation of bilirubin? Bilirubin oxidase Glucuronyl transferase

Glucuronyl transferase

A diabetic patient is prescribed a daily regimen of insulin. Which of the following laboratory procedures would be of most value in determining the degree of glucose control over a 2-month period? Fasting blood glucose 2-hr PP glucose OGTT Glycated hemoglobin Fructosamine

Glycated hemoglobin

Neisseria: A. Gram positive rods, non-spore forming B. Gram positive rods, spore forming C. Gram negative cocci, usually in pairs D. Gram positive cocci, usually in chains E. Gram positive cocci, usually in clusters

Gram negative cocci, usually in pairs

Enterococcus: A. Gram positive rods, non-spore forming B. Gram positive rods, spore forming C. Gram negative cocci, usually in pairs D. Gram positive cocci, usually in chains E. Gram positive cocci, usually in clusters

Gram positive cocci, usually in chains

Staphylococcus: A. Gram positive rods, non-spore forming B. Gram positive rods, spore forming C. Gram negative cocci, usually in pairs D. Gram positive cocci, usually in chains E. Gram positive cocci, usually in clusters

Gram positive cocci, usually in clusters

Corynebacterium: A. Gram positive rods, non-spore forming B. Gram positive rods, spore forming C. Gram negative cocci, usually in pairs D. Gram positive cocci, usually in chains E. Gram positive cocci, usually in clusters

Gram positive rods, non-spore forming

Bacillus: A. Gram positive rods, non-spore forming B. Gram positive rods, spore forming C. Gram negative cocci, usually in pairs D. Gram positive cocci, usually in chains E. Gram positive cocci, usually in clusters

Gram positive rods, spore forming

Pseudomonas aeruginosa A. Bleach odor B. Strawberry odor C. Dirt like odor D. Grape odor E. Mouse nest odor

Grape odor

Protein synthesis, cell growth and division Parathyroid hormone Oxytocin Thyroxine Growth hormone FSH Triiodothyronine Calcitonin ACTH LH Aldosterone TSH ADH

Growth hormone

Which two conditions can "physiologically" elevate serum alkaline phosphatase? Growth, third trimester of pregnancy Obstructive jaundice, Biliary cirrhosis

Growth, third trimester of pregnancy

The screening test used for PKU, that uses dried blood on filter disks is:

Guthrie test

Which feature distinguishes Erysipelothrix rhusiopathiae from other clinically significant non-spore forming, gram positive bacilli? A. Tumbling motility B. Beta hemolysis C. More pronounced motility at 25˚C than 37˚C D. H2S production

H2S production

The lipoprotein that gather up excess cholesterol for transport back to the liver is:

HDL

Which of the following bacteria is associated with chancroid? A. Gardnerella vaginalis B. Haemophilus ducreyi C. Neisseria gonorrhoeae D. Haemophilus influenzae aegypticus

Haemophilus ducreyi

An organism that exhibits the satellite phenomenon around colonies of staphylococci is: A. Haemophilus influenzae B. Neisseria meningitidis C. Neisseria gonorrhoeae D. Klebsiella pneumoniae

Haemophilus influenzae

The bacteria most often responsible for acute epiglottitis is: A. Bordatella pertussis B. Haemophilus influenzae C. Haemophilus aphrophilus D. Enterococcus faecalis

Haemophilus influenzae

The reason carbon monoxide is so toxic is because it:

Has 250 time the affinity of oxygen for hemoglobin binding sites

Iron is physiologically active only in the ferrous form in: Hemoglobin Ferritin

Hemoglobin

The following results are obtained on a 3-year-old boy with sudden severe hemorrhagic problems: Bleeding time Normal PT Normal APTT Prolonged APTT 1:1 Mixing Study Corrected Platelet aggregation Normal with ristocetin, ADP, collagen, and epinephrine These clinical manifestations and lab results are consistent with: A. Aspirin B. Von Willebrand's disease C. Hemophilia A D. Factor VII deficiency

Hemophilia A

Concerning specimen requirements for total serum calcium determination, which of the following anticoagulants is acceptable?

Heparin

Which of the following is the reference method for measuring serum glucose?

Hexokinase

The M:E ratio in chronic myelocytic leukemia is usually: A. Normal B. High C. Low D. Variable

High

In rickets patients, typical findings include all of the following EXCEPT: High alkaline phosphatase Low serum calcium High PTH High serum phosphate

High alkaline phosphatase

A person suspected of having metabolic alkalosis would have which of the following laboratory findings?

High pH, high HCO3

Which of the following is characteristic of Clostridium perfringens? A. Lecithinase positive B. Lipase positive C. Motile D. Esculin hydrolysis positive

Lecithinase positive

A completely sickled cell (drepanocyte) is most commonly seen in which of these conditions? A. Homozygous HbSS B. Heterozygous HbSA C. Double heterozygous HbSC D. Hb S with hereditary persistence of fetal hemoglobin

Homozygous HbSS

What is the reagent used in the catalase test? A. Rubbing Alcohol B. 0.85% Saline C. Distilled water D. Hydrogen peroxide

Hydrogen peroxide

Which of the following is the primary mechanism of compensation for metabolic acidosis?

Hyperventilation

Hyperparathyroidism is most consistently associated with: Hypophosphatemia Hypocalcemia

Hypophosphatemia

Which factors are consumed in clotting and therefore absent in serum? A. I, V, VIII, XIII B. I, II, V, VIII, XIII C. II, VII, IX, X D. VIII, IX, XI, XII

I, V, VIII, XIII

Coagulation factors affected by Coumadin (warfarin) drugs are: A. VIII, IX, and X B. I, II, V, and VII C. II, VII, IX, and X D. II, V, and VII

II, VII, IX, and X

Coagulation factors affected by Coumadin (warfarin) drugs are: A. VIII, IX, and X B. I, II, V, and VII C. II, VII, IX, and X D. II, V, and VII

II, VII, IX, and X

Which of the following methods is commonly used for potassium measurement? ISE Spectrometer

ISE

What is the diagnosis of a child who has onset of bleeding signs and symptoms, while having a normal CBC, a normal physical examination, and no family history of hemorrhagic abnormalities or thrombocytopenia? A. TTP B. ITP C. HUS D. May-Hegglin Anomaly

ITP

What is the diagnosis of a child who has onset of bleeding signs and symptoms, while having a normal CBC, a normal physical examination, and no family history of hemorrhagic abnormalities or thrombocytopenia? A. TTP B. ITP C. HUS D. May-Hegglin Anomaly

ITP

All of the following have been linked to excess erythrocyte destruction except: A. IgG alloantibodies B. IgE alloantibodies C. Drugs D. IgM autoimmune antibodies

IgE alloantibodies

Which of the following is true regarding the mechanism of warm-reactive autoimmune hemolytic anemia (WAHA)? A. IgG autoantibodies cause intravascular hemolysis B. IgG autoantibodies cause extravascular hemolysis C. IgM autoantibodies cause immune complex formation D. IgM alloantibodies cause complement activation

IgG autoantibodies cause extravascular hemolysis

All of the following actions would be appropriate for the investigation of an initially prolonged PT test except? A. Check for clots in the sample B. Check patient history C. Immediately cancel the test and request a new sample D. Check the fill level of the tube

Immediately cancel the test and request a new sample

The most useful classification system for acute lymphoid leukemia (ALL) based is based on: A. Sudan Black B staining B. Periodic acid-Schiff staining C. Immunophenotyping D. Morphology

Immunophenotyping

Patients with Cushing's syndrome exhibit: Increased cortisol production Serum cortisol concentrations less than 1 mg/d

Increased cortisol production

Which one of the following laboratory results is consistent with a diagnosis of Graves disease? Decreased free T3 Increased free T4

Increased free T4

Major actions of angiotensin II include: Increased vasoconstriction Decreased adrenal secretion of aldosterone

Increased vasoconstriction

In a patient with suspected primary hyperthyroidism associated with Graves' disease, one would expect the following laboratory serum results: T4 _________, TSH _________.

Increased, decreased

Factors commonly involved in producing anemia in patients with chronic renal disease include: A. Marrow hypoplasia B. Ineffective erythropoiesis C. Vitamin B12 deficiency D. Increased erythropoietin production

Ineffective erythropoiesis

Why are infants with beta-thalassemia major not ill until approximately 6 months of age? A. Infants have less need for hemoglobin because of their small body size B. Infants have less need for hemoglobin because of their smaller lung capacity C. Infants have a high red count and thus higher hemoglobin from a higher than normal concentration of erythropoietin D. Infants are protected by their high concentration of fetal hemoglobin.

Infants are protected by their high concentration of fetal hemoglobin

Why are infants with beta-thalassemia major not ill until approximately 6 months of age? A. Infants are protected by their high concentration of fetal hemoglobin. B. Infants have less need for hemoglobin because of their small body size. C. Infants have less need for hemoglobin because of their smaller lung capacity. D. Infants have a high red count and thus higher hemoglobin from a higher than normal concentration of erythropoietin.

Infants are protected by their high concentration of fetal hemoglobin.

A small gram negative rod isolated from an eye culture has the following test results: X factor requirement: Yes V factor requirement: Yes This organism is most probably Haemophilus: A. Influenzae B. Parainfluenzae C. Haemolyticus D. Parahaemolyticus

Influenzae

The hormone responsible for entry of glucose into the cells is:

Insulin

Absorption of vitamin B12 in the stomach and gut requires the presence of:

Intrinsic factor

POCT tests classified as waived by government standards include all of the following EXCEPT:

Ionized calcium

Blood specimens drawn early in the morning are recommended for measuring serum iron and TIBC because: Most people's diet is rich in iron Iron exhibits a diurnal cyclic variation Iron in serum goes into the cells after a meal The premise is incorrect; iron specimens do not need to be fasting

Iron exhibits a diurnal cyclic variation

Urea: Is synthesized in the liver from the breakdown of nucleic acids Is synthesized from CO2 and ammonia Can produce Reye's syndrome when present in high amounts in blood Is increased in serum during acute episodes of gout

Is synthesized from CO2 and ammonia

Which of the statements below regarding the metabolism of bilirubin is true? It is a product of prophyrin metabolism It is produced from the destruction of RBCs

It is produced from the destruction of RBCs

Why is glucose-6-phosphate dehydrogenase (G6PD) important for normal red cell survival? A. Hemoglobin oxygen affinity is increased in its absence. B. It is required to regenerate reduced glutathione. C. It is required for insertion of iron into the protoporphyrin ring to form heme. D. Alpha chains are produced in excess in its absence.

It is required to regenerate reduced glutathione.

The most appropriate screening test for detecting hemoglobin S is: A. Osmotic fragility B. Dithionite solubility C. Kleihauer-Betke D. Heat instability test

Kleihauer-Betke

Maturation of follicles, ovulation, production of estrogen, progesterone and testosterone Parathyroid hormone Oxytocin Thyroxine Growth hormone FSH Triiodothyronine Calcitonin ACTH LH Aldosterone TSH ADH

LH

This cardiac protein rises at 12 to 18 hours, peaks at 48-72 hours, and returns to normal after 6-10 days Lactate Dehydrogenase CK-MB CK CK-MM

Lactate Dehydrogenase

What MINIMUM level of a particular factor will cause the aPTT test to become prolonged? A. Less than 40% B. Less than 50% C. Less than 60% D. Less than 70%

Less than 40%

A patient admitted to the hospital for ongoing fever produces the following laboratory results: RBC count: 3.56 x 1012/L WBC count: 57.5 x 109/L Platelet count: 375,000/uL Differential count: 3 blasts, 10 myelocytes, 6 metamyelocytes, 12 bands, 64 segs, 4 lymphocytes, and 1 monocyte LAP score = 155 Which of the following conditions correlates closely with this patient's results? A. Leukemoid Reaction B. Chronic Myelogenous Leukemia C. Genetic translocation (9;22)(q34;q11) D. Paroxysmal Nocturnal Hemoglobinuria

Leukemoid Reaction

A patient admitted to the hospital for ongoing fever produces the following laboratory results: RBC count: 3.56 x 1012/L WBC count: 57.5 x 109/L Platelet count: 375,000/uL Differential count: 3 blasts, 10 myelocytes, 6 metamyelocytes, 12 bands, 64 segs, 4 lymphocytes, and 1 monocyte LAP score = 155 Which of the following conditions correlates closely with this patient's results? A. Leukemoid Reaction B. Chronic Myelogenous Leukemia C. Genetic translocation (9;22)(q34;q11) D. Paroxysmal Nocturnal Hemoglobinuria Leukemoid Reaction

Leukemoid Reaction

The leukocyte alkaline phosphatase activity is increased in: A. Erythroleukemia B. Leukemoid reaction C. Chronic granulocytic leukemia D. Acute granulocytic leukemia

Leukemoid reaction

Which of the following bacteria exhibit a tumbling motility? A. Corynebacterium diphtheriae B. Bacillus cereus C. Listeria monocytogenes D. Corynebacterium jeikeium

Listeria monocytogenes

The isolation of a small gram-positive, catalase-positive rod with a narrow zone of beta hemolysis isolated from blood or CSF should be used as strong presumptive evidence for: A. Listeriosis B. Diphtheria C. Pertussis D. Tetanus

Listeriosis

Which of the following is used primarily for the treatment of manic-depression:

Lithium

Plasma proteins are synthesis primarily in the: kidney liver pancreas

Liver

What would be a PROBABLE diagnosis for a habitual alcoholic with slight elevations in the PT and APTT tests? A. Lupus inhibitor B. DIC C. Platelet antibody D. Liver disease

Liver disease

The Jaffe alkaline picrate reaction for creatinine can be made more specific for creatinine with the use of:

Lloyd's reagent

To differentiate Corynebacterium jeikeium and Corynebacterium urealyticum from other Corynebacterium species, the use of which agar will enhance their growth? A. Loeffler B. TCBS C. Tween 80 D. Middlebrook

Loeffler

A patient presents with Addison's disease. Serum sodium and potassium analyses are done. The results would reveal: Normal sodium, low potassium levels Low sodium, low potassium levels Low sodium, high potassium levels High sodium, low potassium levels

Low sodium, high potassium levels

One specific agar for Mycobacterium spp. is: A. Regan-Lowe B. Cystine tellurite C. Lowenstein-Jennings D. BCYE

Lowenstein-Jennings

What test is used along with the mean cell volume (MCV) to morphologically classify anemias? A. red blood count B. hemoglobin C. hematocrit D. MCHC

MCHC

Skeletal muscle tissue contains which of the following CK isoenzymes?

MM+MB

Urine culture should be plated to: A. Sheep blood agar and chocolate agar B. Sheep blood agar, CNA/PEA, and thioglycollate broth C. MacConkey agar and Sheep blood agar D. MacConkey agar, sheep blood agar, chocolate agar, and CNA/PEA

MacConkey agar and Sheep blood agar

Lithium therapy is used widely in the treatment of:

Manic depression

Which of the following is the selective media used specifically for growing Staphylococcus aureus? A. Mannitol B. Tween 80 C. Middlebrook D. Loeffler

Mannitol

A child that has skin that smells like burnt sugar and is suffering from muscle rigidity, stupor and respiratory irregularities.

Maple Syrup Urine Disease

Which of the following platelet responses is most likely associated with Glanzmann's thrombasthenia? A. Decreased platelet aggregation to ristocetin B. Defective ADP release, normal response to ADP C. Decreased amount of ADP in platelets D. Markedly decreased aggregation to epinephrine, ADP, and collagen

Markedly decreased aggregation to epinephrine, ADP, and collagen

What is the gold standard methodology for Toxicology testing?

Mass Spectrometry

Which of the following anomalies is an autosomal dominant disorder characterized by irregularly sized inclusions in polymorphonuclear neutrophils, abnormal giant platelets, and often thrombocytopenia? A. Pelger-Huet B. Chediak-Higashi C. Alder-Reilly D. May-Hegglin

May-Hegglin

Which of the following is associated with Chediak-Higashi syndrome? A. Membrane defect of lysosomes B. Dohle-like bodies and giant platelets C. Two-lobed neutrophils D. Mucopolysaccharidosis

Membrane defect of lysosomes

Select the form of hemoglobin that contains iron in the ferric rather than the ferrous state. A. Methemoglobin B. Deoxyhemoglobin C. Myoglobin D. Carboxyhemoglobin

Methemoglobin

What is the pathogenesis of microangiopathic hemolytic anemia (MAHA)? A. Microthrombi and fibrin formed on damaged endothelial cells trap and break red cells. B. Chemicals or heat destroy red blood cells. C. The spleen sequesters red cells in an attempt to remove abnormal inclusions. D. Antibodies that activate complement are formed and destroy the red cell membrane.

Microthrombi and fibrin formed on damaged endothelial cells trap and break red cells

Which of the following best describes Hepatitis A?

Mode of transmission from stool

What is the primary clinical utility of measuring CEA?

Monitoring for recurrence of cancer

Haemophilus influenzae A. Bleach odor B. Strawberry odor C. Dirt like odor D. Grape odor E. Mouse nest odor

Mouse nest odor

Which of the following is characteristic of platelet disorders? A. Deep muscle hemorrhages B. Retroperitoneal hemorrhages C. Mucous membrane hemorrhages D. Severely prolonged clotting times

Mucous membrane hemorrhages

Which of the following is characteristic of platelet disorders? A. Deep muscle hemorrhages B. Retroperitoneal hemorrhages C. Mucous membrane hemorrhages D. Severely prolonged clotting times

Mucous membrane hemorrhages

Creatinine excretion correlates best with which of the following parameters: Muscle mass Body weight

Muscle mass

All of the following are capable of causing tuberculosis except: A. Mycobacterium tuberculosis B. Mycobacterium avium C. Mycobacterium bovis D. Mycobacterium africanum

Mycobacterium avium

A scrotochromogen often found as a contaminant but rarely, if ever, as a pathogen is: A. Mycobacterium gordonae B. Mycobacterium scrofulaceum C. Mycobacterium chelonae D. Mycobacterium marinum

Mycobacterium marinum

Beta-lactam antimicrobials comprise the penicillins, cephalosporins, carbapenams, and monbactams. These antimicrobials are bactericidal to susceptible bacteria. Their therapeutic application is recommended for all the following etiologic agents EXCEPT: A. Mycoplasma B. E.coli C. Non-penicillinase-producing strains of Staph aureus D. Streptococcus agalactiae

Mycoplasma

Which of the following agents lacks a cell wall and is the smallest known free-living form? A. Chlamydia B. Mycoplasma C. cRickettsia D. Coxiella

Mycoplasma

Which organism is the etiologic agent of primary atypical pneumonia, often referred to as "walking pneumonia"? A. Streptococcus pneumoniae B. Chlamydophila pneumoniae C. Klebsiella pneumoniae D. Mycoplasma pneumoniae

Mycoplasma pneumoniae

What is the last granulocyte in the maturation sequence able to undergo mitosis? A. Band B. Promyelocytes C. Myelocyte D. Myeloblast

Myelocyte

In which of the following disease states are teardrop cells and abnormal platelets most characteristically seen? A. Chronic myelocytic leukemia B. Multiple myeloma C. Uremia D. Myeloid metaplasia

Myeloid metaplasia

Proteus mirabilis is indole: A. Negative B. Positive

Negative

All the following test results are characteristic of DIC except: A. Decreased fibrinogen concentration B. Negative test for degradation products C. Decreased platelet count D. Prolonged PT

Negative test for degradation products

All the following test results are characteristic of DIC except: A. Decreased fibrinogen concentration B. Negative test for degradation products C. Decreased platelet count D. Prolonged PT

Negative test for degradation products

Which patient group is most susceptible to bleeding because of a deficiency of vitamin K-dependent proteins? A. Pregnant women B. Those on antibiotics C. Those with poor diets D. Newborns

Newborns

Which patient group is most susceptible to bleeding because of a deficiency of vitamin K-dependent proteins? A. Pregnant women B. Those on antibiotics C. Those with poor diets D. Newborns

Newborns

A bone marrow slide shows foam cells ranging from 20 to 100 µm in size with vacuolated cytoplasm containing sphingomyelin and is faintly PAS positive. This cell type is most characteristic of: A. Gaucher's disease B. Myeloma with Russell bodies C. Di Guglielmo disease D. Niemann-Pick disease

Niemann-Pick disease

A patient has a plasma myoglobin level of 10µg/L at admission. Three hours later the myoglobin is 14µ/L and the Troponin I is 0.04µg/L (reference range 0-0.04µg/L). These results are consistent with which condition? No evidence of myocardial or skeletal muscle injury Acute MI

No evidence of myocardial or skeletal muscle injury

How would the RBCs appear on the peripheral blood smear if the red cell indices obtained on a patient are as follows: MCV 88.5 f1 MCH 30.2 pg MCHC 33.1 % A. Hypochromic, microcytic B. Normochromic, microcytic C. Normochromic, normocytic D. Hypochromic, normocytic

Normochromic, normocytic

Which of the following would not be represented in the usual classification of anemia: A. Microcytic, hypochromic B. Normocytic, normochromic C. Normocytic, hyperchromic D. Macrocytic

Normocytic, hyperchromic

RBC indices obtained on a patient are as follows: MCV = 88 fL MCH = 30 pg MCHC = 34 g/dL A. Microcytic, hypochromic B. Microcytic, normochromic C. Normocytic, normochromic D. Normocytic, hypochromic

Normocytic, normochromic

Thayer-Martin has 4 antibiotics, match up what each antibiotic inhibits. What inhibits yeast? A. Colistin B. Trimethoprim C. Nystatin D. Vancomycin

Nystatin

Activated forms of normal cellular genes are termed:

Oncogenes

Interpret the following results: Cholesterol: 178 mg/dL Triglycerides: 233 mg/dL HDL: 52 mg/dL LDL: 79.4 mg/dL All results are normal All results are abnormal Only the Triglyceride is abnormal The LD and HDL are abnormal

Only the Triglyceride is abnormal

Haemophilus influenzae is most likely considered normal flora in the: A. Oropharynx B. Female genital tract C. Large intestine D. Small intestine

Oropharynx

The most appropriate screening test for hereditary spherocytosis is: A. Osmotic fragility B. Sucrose hemolysis C. Heat instability test D. Kleihauer-Betke

Osmotic fragility

Creatinine is formed from the:

Oxidation of creatine

What would be the expected screening test results for a patient with a fibrin stabilizing factor deficiency? A. PT prolonged B. APTT prolonged C. PT and APTT prolonged D. PT and APTT normal

PT and APTT normal

What would be the expected screening test results for a patient with a fibrin stabilizing factor deficiency? A. PT prolonged B. APTT prolonged C. PT and APTT prolonged D. PT and APTT normal

PT and APTT normal

Which of the following tests is increased in a patient with severe hemophilia A? A. PTT B. PT C. Factor VIII assay D. TT

PTT

Progesterone: Is produced in adult testes and is responsible for genital development, beard growth, muscle development, and sexual drive Is produced by the placenta during pregnancy, with highest levels seen at conception and then steadily decreasing to non-detectable levels at term Is lowest is serum during the luteal phase of the menstrual cycle and highest during the follicular phase Parallels activity of the corpus luteum by rapidly increasing following ovulation and then abruptly falling to initial low concentrations prior to the onset of menstruation

Parallels activity of the corpus luteum by rapidly increasing following ovulation and then abruptly falling to initial low concentrations prior to the onset of menstruation

Which of the following organisms is still susceptible to penicillin drugs? A. Pasteurella multocida B. Nocardia C. Pseudomonas aeruginosa D. Eikenella corrodens

Pasteurella multocida

The drug of choice for most streptococcal infections is: A. Vancomycin B. Streptomycin C. Penicillin D. Septra

Penicillin

The bond that is created when the carboxyl group of one amino acid and the amino group of another is

Peptide bond

In which malaria species do you see a characteristic band in the trophozoite stage? A. Plasmodium vivax B. Plasmodium ovale C. Plasmodium falciparum D. Plasmodium malariae

Plasmodium malariae

The final result of primary hemostasis is: A. Platelet plug B. Constriction of a blood vessel C. A blood clot D. Promotion of bleeding to cleanse a wound

Platelet plug

Identify the platelet receptor and plasma protein required for normal platelet adhesion. A. Platelet receptor GP IV Plasma protein factor VIII B. Platelet receptor PF-4 Plasma protein factor V C. Platelet receptor GP IIb/IIIa Plasma protein fibrinogen D. Platelet receptor GP Ib/IX/V Plasma protein von Willebrand factor

Platelet receptor GP Ib/IX/V Plasma protein von Willebrand factor

Identify the platelet receptor and plasma protein required for normal platelet adhesion. A. Platelet receptor GP IV Plasma protein factor VIII B. Platelet receptor PF-4 Plasma protein factor V C. Platelet receptor GP IIb/IIIa Plasma protein fibrinogen D. Platelet receptor GP Ib/IX/V Plasma protein von Willebrand factor

Platelet receptor GP Ib/IX/V Plasma protein von Willebrand factor

A bleeding time is used to evaluate the activity of: A. Platelets B. Prothrombin C. Labile factor D. Factor XIII

Platelets

A bleeding time is used to evaluate the activity of: A. Platelets B. Prothrombin C. Labile factor D. Factor XIII

Platelets

What is the last erythrocyte precursor capable of cell division? A. Orthochromic Normoblast B. Reticulocyte C. Basophilic Normoblast D. Polychromatic Normoblast

Polychromatic Normoblast

Diabetes insipidus is a disease characterized by: Polydipsia, oliguria, and urine with a low specific gravity Polydipsia, polyuria, and urine with a high specific gravity Polydipsia, polyuria, and urine with low specific gravity Polydipsia, oliguria, and urine with a specific gravity

Polydipsia, polyuria, and urine with low specific gravity

E. coli is indole: A. Negative B. Positive

Positive

Which of the following conditions would be suggested by a jaundiced patient experiencing a marked rise in alkaline phosphatase, conjugated bilirubin, and a slight rise in ALT:

Post-hepatic obstruction

What is the most common site for bone marrow aspiration in an adult? A. Sternum B. Tibia C. Posterior iliac crest D. Ribs

Posterior iliac crest

An electrolyte panel (lytes, chem-4) consists of: Potassium, Sodium, Chloride, BUN Potassium, Sodium, Chloride, Carbon dioxide

Potassium, Sodium, Chloride, Carbon dioxide

Which of the following factors is Fletcher factor? A. HMWK B. Factor III C. Prekallikrein D. Factor XIII

Prekallikrein

Results on a patient presenting with sudden severe hemorrhagic problems are as follows: Bleeding time Normal PT Normal APTT Prolonged APTT 1:1 Mixing Study No correction These clinical manifestations and lab results are consistent with: A. Coumadin therapy B. Von Willebrand disease C. Hemophilia A D. Presence of a circulating inhibitor

Presence of a circulating inhibitor

Results on a patient presenting with sudden severe hemorrhagic problems are as follows: Bleeding time Normal PT Normal APTT Prolonged APTT 1:1 Mixing Study No correction These clinical manifestations and lab results are consistent with: A. Coumadin therapy B. Von Willebrand disease C. Hemophilia A D. Presence of a circulating inhibitor

Presence of a circulating inhibitor

The serum TSH level is decreased in: Primary hyperthyroidism Secondary hyperthyroidism

Primary hyperthyroidism

The TRH (thyrotropin releasing hormone) stimulation test is useful in differentiating hypothalamic hypothyroidism from: Primary hypothyroidism Hashimoto's thyroiditis Pituitary hypothyroidism Sub-clinical hypothyroidism

Primary hypothyroidism

The structure that represents the sequence of a polypeptide chain is the: Primary structure Tertiary structure

Primary structure

The structure that represents the sequence of polypeptide chain is the: Primary structure Quaternary structure

Primary structure

Which of the following readies the uterus for implantation of an embryo?

Progesterone

Pituitary adenomas that secrete _______ are by far the most common form of hyperpituitarism. Growth hormone Prolactin ACTH TSH ADH

Prolactin

What is the first feature of nephrotic syndrome that results in manifestation of the other four classic signs?

Proteinuria

A gram negative rod was isolated from a wound infection caused by a bite from a pet cat. The following characteristic reactions were seen: Oxidase: Positive Glucose: Fermentative Catalase: Positive Motility: Negative MacConkey agar: No growth Which of the following is the most likely organism? A. Pseudomonas aeruginosa B. Pasteurella multocida C. Aeromonas hydrophila D. Vibrio cholera

Pseudomonas aeruginosa

Which of the following bacteria is a gram negative rod with a metallic sheen and grape-like odor? A. Alcaligenes faecalis B. Pseudomonas aeruginosa C. Burkholderia cepacia D. Stenotrophomonas maltophilia

Pseudomonas aeruginosa

Which of the following bacteria is associated with swimmer's ear? A. Alcaligenes faecalis B. Pseudomonas aeruginosa C. Burkholderia cepacia D. Stenotrophomonas maltophilia

Pseudomonas aeruginosa

An organism described as oxidase-positive, TSI slant with an alkaline/no change, good growth at 42 degrees C, and producing bright bluish green pigment on Mueller-Hinton agar is most likely: A. Pseudomonas aeruginosa. B. Ralstonia pickettii. C. Burkholderia mallei. D. Burkholderia pseudomallei.

Pseudomonas aeruginosa.

A chemistry analyzer that can perform multiple analyses on a single specimen during one run is referred to as a:

Random access analyzer

A chemistry analyzer that can perform multiple analyses on a single specimen during one run is referred to as a: A. Batch analyzer B. Random access analyzer C. Continuous-flow analyzer D. Chromatographic analyzer

Random access analyzer

A thrombosis risk testing profile is ordered on a patient being treated with warfarin for a blood clot in the leg. Select the best course of action. A. Recommend the testing be postponed until the warfarin is discontinued B. Reject the order C. Perform only the tests for protein C and protein S D. Run the tests immediately

Recommend the testing be postponed until the warfarin is discontinued

Why does Plasmodium falciparum infection result in the most serious hemolysis of the various forms of malaria? A. Only reticulocytes are invaded B. Reticulocytes and red blood cells <10 days old are invaded C. Only mature red blood cells are invaded D. Red blood cells of all ages are invaded

Red blood cells of all ages are invaded

An arterial blood sample is received in the laboratory 45 minutes after collection with a bubble in the syringe. What should the MLT do?

Reject the sample because the pCO2 will be falsely decreased

Secondary hyperparathyroidism is often the result of: Pituitary disease Renal disease

Renal disease

Which of the following conditions will cause an increased anion gap? Diarrhea Hypoaldosteronism Hyperkalemia Renal failure

Renal failure

Which of the following conditions will cause an increased anion gap? Renal failure Diarrhea

Renal failure

Which of the following best describes Hepatitis D? Mode of transmission from stool Requires a co-infection with HBV

Requires a co-infection with HBV

Which of the following is characteristic of type 1 diabetes mellitus?

Requires insulin replacement to prevent ketosis

The thyrotropin releasing hormone (TRH) stimulation test rules out the diagnosis of mild or subclinical hyperthyroidism if TRH infusion causes: No rise in plasma TSH Rise in plasma TSH

Rise in plasma TSH

Yersinia pestis: A. Boxcar shape B. Cat eyes C. Chinese letters D. Cluster of grapes E. Branching F. Safety pins G. String of pearl

Safety pins

The gram-negative bacillus that can be described as oxidase-negative, nitrate-positive, indole-negative, citrate-positive, methyl red-positive, urease-negative, H2S-positive is: A. Klebsiella pneumoniae B. Salmonella enteritidis C. Escherichia coli D. Shigella sonnei E. Proteus vulgaris

Salmonella enteritidis

The presence of excessive rouleaux formation on a blood smear is often accompanied by an increased: A. Reticulocyte count B. Sedimentation rate C. Hematocrit D. Erythrocyte count

Sedimentation rate

Match the category of agar for each of the following. More than one choice can be used for each type of agar. _______ CNA A. Selective B. Supportive C. Enrichment D. Differential

Selective

Match the category of agar for each of the following. More than one choice can be used for each type of agar. _______ PEA A. Selective B. Supportive C. Enrichment D. Differential

Selective

Match the category of agar for each of the following. More than one choice can be used for each type of agar. _______ Hektoen Enteric (HE) A. Selective B. Supportive C. Enrichment D. Differential

Selective, Differential

Match the category of agar for each of the following. More than one choice can be used for each type of agar. _______ MacConkey A. Selective B. Supportive C. Enrichment D. Differential

Selective, Differential

Match the category of agar for each of the following. More than one choice can be used for each type of agar. _______ XLD A. Selective B. Supportive C. Enrichment D. Differential

Selective, Differential

Match the category of agar for each of the following. More than one choice can be used for each type of agar. _______ Thayer-Martin A. Selective B. Supportive C. Enrichment D. Differential

Selective, Enrichment

Which of the following bacteria has a red pigment on MacConkey agar? A. E. coli B. Serratia marcescens C. Pseudomonas aeruginosa D. Klebsiella oxytoca

Serratia marcescens

Which of the following represents characteristic features of iron metabolism in patients with anemia of a chronic disorder? A. Serum iron is normal, transferrin saturation is normal, TIBC is normal B. Serum iron is increased, transferrin saturation is increased, TIBC is normal or slightly increased C. Serum iron is normal, transferrin saturation is markedly increased, TIBC is normal D. Serum iron is decreased, transferrin saturation is decreased, TIBC is normal or decreased

Serum iron is decreased, transferrin saturation is decreased, TIBC is normal or decreased

In an uncomplicated case of severe iron deficiency anemia, which of the following sets represents the typical pattern of results? Serum Iron Serum TIBC % Saturation Marrow % Sideroblasts Marrow Iron Stores Serum Ferritin Set 1 Decreased Increased Decreased Decreased Increased Increased Set 2 Decreased Decreased Decreased Decreased Decreased Decreased Set 3 Decreased Increased Decreased Decreased Decreased Decreased Set 4 Decreased Decreased Increased Increased Increased Increased A. Set 1 B. Set 2 C. Set 3 D. Set 4

Set 3

A test's "negativity in the absence of the disease" is termed its: Specificity Predictive value Accuracy Sensitivity Efficiency

Specificity

Rose Gardener's disease is caused by: A. Histoplasma capsulatum B. Sporothrix schenckii C. Blastomyces dermatitidis D. Coccidioides immitis

Sporothrix schenckii

The coefficient of variation (CV) is: The square root of the standard deviation Standard deviation expressed as % of the average of the values used to calculate the standard deviation Standard deviation expressed as % of the range of values used to calculate the standard deviation Standard deviation expressed as % of the correlation coefficient Summation of duplicate differences expressed as % of the mean v

Standard deviation expressed as % of the average of the values used to calculate the standard deviation

Choose the group of bacteria that is described as catalase-positive, gram-positive cocci that grow facultatively anaerobic and that form grapelike clusters. A. Neisseria B. Staphylococcus C. Micrococcus D. Streptococcus

Staphylococcus

This virulent and important human pathogen can also be recovered from the nares, perineum, and other skin sites of healthy people, especially in the hospital setting. A. Micrococcus luteus B. Staphylococcus intermedius C. Staphylococcus aureus D. All Micrococcaceae

Staphylococcus aureus

Of the following bacteria, the most frequent cause of prosthetic heart valve infections occurring within two to three months after surgery is: A. Streptococcus pneumonia B. Streptococcus pyogenes C. Staphylococcus aureus D. Staphylococcus epidermidis

Staphylococcus epidermidis

Which bacteria is associated with dog bites? A. Staphylococcus aureus B. Staphylococcus intermedius C. Staphylococcus hominis D. Staphylococcus lugdunensis

Staphylococcus intermedius

Which of the following bacteria is PYR positive? A. Staphylococcus aureus B. Staphylococcus hominis C. Staphylococcus intermedius D. Staphylococcus saprophyticus

Staphylococcus intermedius

The staphylococcus that is more likely to cause uncomplicated urinary tract infections in nonhospitalized hosts, especially sexually active young women, is: A. Staphylococcus saprophyticus B. Staphylococcus aureus C. Staphylococcus epidermidis D. Staphylococcus intermedius

Staphylococcus saprophyticus

Which of the following bacteria is a gram negative rod with an ammonia odor? A. Alcaligenes faecalis B. Pseudomonas aeruginosa C. Burkholderia cepacia D. Stenotrophomonas maltophilia

Stenotrophomonas maltophilia

Alcaligenes faecalis A. Bleach odor B. Strawberry odor C. Dirt like odor D. Grape odor E. Mouse nest odor

Strawberry odor

Choose the genus that is best described as a catalase-negative, gram-positive coccus that is a facultative anaerobe and that forms chains. A. Streptococcus B. Stomatococcus C. Staphylococcus D. Micrococcus

Streptococcus

Match the Lancefield group with the corresponding Streptococcus species. Group B A. Streptococcus bovis B. Streptococcus pyogenes C. Streptococcus agalactiae D. Streptococcus pneumoniae

Streptococcus agalactiae

Match the Lancefield group with the corresponding Streptococcus species. Group D A. Streptococcus bovis B. Streptococcus pyogenes C. Streptococcus agalactiae D. Streptococcus pneumoniae

Streptococcus pneumoniae

Match the Lancefield group with the corresponding Streptococcus species. None A. Streptococcus bovis B. Streptococcus pyogenes C. Streptococcus agalactiae D. Streptococcus pneumoniae

Streptococcus pneumoniae

The Optochin disk is used for the identification of: A. Haemophilus influenza B. Streptococcus pyogenes C. Staphylococcus hyicus D. Streptococcus pneumoniae

Streptococcus pneumoniae

Match the Lancefield group with the corresponding Streptococcus species. Group A A. Streptococcus bovis B. Streptococcus pyogenes C. Streptococcus agalactiae D. Streptococcus pneumoniae

Streptococcus pyogenes

Which toxin produced by Streptococcus pyogenes will cause the hemolysis on the agar plate? A. Erythrogenic toxin B. Hyaluronidase C. M protein D. Streptolysin S

Streptolysin S

Which of the following tests is able to differentiate Vibrio spp. from Aeromonas and Plesiomonas? A. Catalase B. Oxidase C. String D. Urea

String

Streptococcus: A. Boxcar shape B. Cat eyes C. Chinese letters D. Cluster of grapes E. Branching F. Safety pins G. String of pearls

String of pearls

The qualitative hemoglobinopathies cause disease by producing hemoglobin chains that are: A. Structurally altered B. Missing C. Produced in inadequate amounts D. Too long

Structurally altered

The most appropriate screening test for paroxysmal nocturnal hemoglobinuria is: A. Heat instability test B. Sucrose hemolysis C. Osmotic fragility D. Dithionite solubility

Sucrose hemolysis

Match the category of agar for each of the following. More than one choice can be used for each type of agar. _______ Sheep Blood Agar A. Selective B. Supportive C. Enrichment D. Differential

Supportive, Differential

Match the category of agar for each of the following. More than one choice can be used for each type of agar. _______ Chocolate A. Selective B. Supportive C. Enrichment D. Differential

Supportive, Enrichment

Match the category of agar for each of the following. More than one choice can be used for each type of agar. _______ Thioglycollate broth A. Selective B. Supportive C. Enrichment D. Differential

Supportive, Enrichment

Mature T cells with cerebriform, clefted nuclei found in the skin and peripheral blood describe: A. hairy cells B. prolymphocytes C. lymphoblasts D. Sézary cells

Sézary cells

A 15-year-old boy presents complaining of severe headaches; he has a white count of 76.0 x 109/L, platelet count of 55 x 109/L, and a hematocrit of 33%. When a white cell differential is performed, most of the nucleated cells are blasts. They are CD2, CD4, and CD8 positive. What is the most likely diagnosis? A. acute myeloid leukemia without maturation B. immature B cell acute lymphoid leukemia C. acute myeloid leukemia with maturation D. T cell acute lymphoid leukemia

T cell acute lymphoid leukemia

Which formula provides the best estimate of serum transferrin? Serum Fe/TIBC TIBC (µg/dL) x 0.70 = transferrin in mg/dL Percent iron saturation x TIBC (µg/dL)/1.2 + 0.06 mg/dL Serum Fe (µg/dL) x 1.25 = transferrin (µg/dL)

TIBC (µg/dL) x 0.70 = transferrin in mg/dL

The thyroid gland produces all of the following hormones except: TSH Triiodothyronine

TSH

The possible defective sites that correspond to primary, secondary, and tertiary levels of endocrine dysfunction are, respectively: Target gland, pituitary, hypothalamus Pituitary, hypothalamus, target gland

Target gland, pituitary, hypothalamus

An example of a bacterial antibiotic is: A. Chloramphenicol B. Tetracycline C. Tobramycin D. Erythromycin

Tetracycline

Urea is produced from:

The catabolism of proteins and amino acids

Urea is produced from: The catabolism of proteins and amino acids Oxidation of purines Oxidation of pyrimidines The breakdown of complex carbohydrates

The catabolism of proteins and amino acids

As part of a hyperlipidemia screening program, the following results were obtained on a 25-year-old woman 6 hours after eating: Triglycerides 260 mg/dL Cholesterol 120 mg/dL

The cholesterol is normal but the triglycerides are elevated, which may be attributed to the recent meal.

Bioavailablity of a drug refers to:

The fraction of the drug absorbed into the systemic circulation

Three sets of blood cultures were obtained. The aerobic bottle of one set had growth of Staphylococcus epidermidis on the 7-day subculture. This indicates that: A. There was a low-grade bacteremia B. The organism is most likely a contaminant C. The subculture plates were defective D. The subculture should have been done at 5 days

The organism is most likely a contaminant

Which of the following is true regarding a cerebrospinal fluid specimen received for microbiological analysis? A. The specimen should be refrigerated and precessed when possible B. The specimen should be processed immediately C. The specimen shouls be frozen and processed when possible D. The speciment may be held at room temperature and processed any time within 24 hours without loss of organism viability

The specimen should be processed immediately

A drug that relaxes the smooth muscles of the bronchial passages is: Acetaminophen Lithium Phenytoin Theophylline

Theophylline

Beriberi is associated with deficiency of:

Thiamine

The differential meadium often used in the primary isolation and identification of Vibrio is: A. Thiosulfate citrate bile salts-sucrose (TCBS) B. Xylose lysine deoxycholate (XLD) agar C. MacConkey agar D. Eosin nethylene blue (EMB) agar

Thiosulfate citrate bile salts-sucrose (TCBS)

What is the single most common cause of clinically important bleeding? A. Hemophilia B. Qualitative platelet disorders C. Fibrinogen deficiency D. Thrombocytopenia

Thrombocytopenia

Inappropriate formation of platelets or fibrin clots that obstructs blood vessels is called: A. Thrombophilia B. Thrombosis C. Embolus D. Factor V Leiden

Thrombosis

Patients who have a deficiency of protein C or protein S have which type of disorder? A. Bleeding B. Thrombotic C. Bruising D. Hypercalcemia

Thrombotic

Patients who have a deficiency of protein C or protein S have which type of disorder? A. Bleeding B. Thrombotic C. Bruising D. Hypercalcemia

Thrombotic

Patients who have a deficiency of protein C or protein S have which type of disorder? A. Hypercalcemia B. Bruising C. Thrombotic D. Bleeding

Thrombotic

A two-year-old child with a decreased serum T4 is described as being somewhat dwarfed, stocky, and overweight and having coarse features. Of the following, the most informative additional laboratory test would be serum:

Thyroid-stimulating hormone (TSH)

Principle thyroid hormone responsible for metabolism, growth and development Parathyroid hormone Oxytocin Thyroxine Growth hormone FSH Triiodothyronine Calcitonin ACTH LH Aldosterone TSH ADH

Thyroxine

Which of the following proteins binds ferric iron and is increased in Iron-deficiency anemia?

Transferrin

Which of the following tests requires a fasting specimen?

Triglycerides

Mostly formed from the deiodination of t4, responsible for metabolism, growth and development Parathyroid hormone Oxytocin Thyroxine Growth hormone FSH Triiodothyronine Calcitonin ACTH LH Aldosterone TSH ADH

Triiodothyronine

Thayer-Martin has 4 antibiotics, match up what each antibiotic inhibits. What inhibits Proteus? A. Colistin B. Trimethoprim C. Nystatin D. Vancomycin

Trimethoprim

The finding of indigo-blue colored urine is indicative of a defect in the metabolism of: Tyrosine Tryptophan Cystine Phenylalanine

Tryptophan

A patient has a hematocrit of 65%. Which of the following is true related to coagulation testing? A. Tube must be recollected using 0.34 mL of sodium citrate to 4.5 mL of blood B. Tube must be recollected using 0.62 mL of sodium citrate to 4.5 mL of blood C. Hematocrit does not affect coagulation testing D. Tube must be recollected with heparin as the anticoagulant

Tube must be recollected using 0.34 mL of sodium citrate to 4.5 mL of blood

What error was probably made if red cells are seen when attempting to manually count white blood cells on a hemocytometer? A. Diluent was squeezed out and lost when filling the unopette B. Unopette was not allowed to sit for 10 minutes for complete RBC lysis C. The wrong dilution was made D. Unopette was not mixed before charging the hemacytometer

Unopette was not allowed to sit for 10 minutes for complete RBC lysis

A gastroenterologist submits a gastric biopsy from a patient with a peptic ulcer. To obtain presumptive evidence of Helicobacter pylori, a portion of the specimen should be added to which media? A. Urea broth B. Tryptophane C. Hippurate hydrolysis D. PYR

Urea broth

Which of the following tests is used to tell Corynebacterium diphtheriae from other Corynebacterium species? A. Hippurate B. Urea hydrolysis C. Catalase D. CAMP test

Urea hydrolysis

A toxic condition involving a very high serum level of urea and creatinine accompanied by failure of the three main functions of the renal system is referred to as: Acute glomerulonephritis Uremia Azotemia Acute renal failure

Uremia

Chlamydial infections have been implicated in: A. Urethritis and conjunctivitis B. Gastroenteritis and urethritis C. Neonatal pneumonia and gastroenteritis D. Neonatal meningitis and conjunctivitis.

Urethritis and conjunctivitis

Bilirubin is reduced to __________ in the small intestine.

Urobilinogen

In what is a patient with the following coagulation test results most likely deficient? Test Result Reference Range Prothrombin time (PT) 17 seconds 11-15 seconds Partial thromboplastin time 31 seconds 25-37 seconds Thrombin time (TT) 16 seconds 15-21 seconds A. VIII B. X C. VII D. Fibrinogen

VII

Which of the following single-factor inherited deficiencies is most common as the cause for hemophilia? A. V B. IX C. XI D. VIII

VIII

Select the lipoprotein fraction that carries most of the endogenous triglycerides. Chylomicrons VLDL

VLDL

Thayer-Martin has 4 antibiotics, match up what each antibiotic inhibits. What inhibits gram positive? A. Colistin B. Trimethoprim C. Nystatin D. Vancomycin

Vancomycin

Which of the following antimicrobial agents acts by inhibiting cell wall synthesis? A. Vancomycin B. Clindamycin C. Naladixic acid D. Gentamicin

Vancomycin

Which of the following antimicrobial agents acts by inhibiting cell wall synthesis? A. Vancomycin B. Clindimycin C. Naladixic Acid D. Gentamicin

Vancomycin

The selective medium thiosulfate citrate bile salts sucrose (TCBS) agar is especially formulated for isolating what pathogen from stool cultures? A. Vibrio B. Salmonella C. Shigella D. Plesiomonas

Vibrio

Which of the following bacteria has a curved rod appearance? A. Aeromonas spp. B. Chromobacterium violaceum C. Vibrio cholera D. Plesiomonas shigelloides

Vibrio cholera

Aspartate Aminotransferase (AST) and Alanine Aminotransferase (ALT) are both elevated in which of the following diseases?

Viral hepatitis

Aspartate aminotransferase (AST) and alanine aminotransferase (ALT) are both elevated in which of the following diseases?

Viral hepatitis

Aspartate aminotransferase (AST) and alanine aminotransferase (ALT) are both elevated in which of the following diseases? Muscular dystrophy Viral hepatitis Myocardial infarction Obstructive liver disease

Viral hepatitis

Which one of the following vitamins would be least affected by long periods of fat malabsorption? Vitamin D Vitamin C

Vitamin C

A patient is stabilized on warfarin with an international normalized ratio (INR) of 2.7. He develops pneumonia and is admitted to the hospital. His appetite is depressed and he is placed on broad-spectrum antibiotics. At 1 week after the admission, his INR is 5.0. What is the most likely interpretation? A. A new sample should be collected to rule out a clotted specimen B. A partial thromboplastin time (PTT) should be performed to rule out heparin administration C. The patient should be given more warfarin D. Vitamin K is reduced, increasing the affects of warfarin

Vitamin K is reduced, increasing the affects of warfarin

The following lab results were obtained for a patient with an inherited autosomal dominant trait: Bleeding time Prolonged Platelet adhesiveness Abnormal PT Normal APTT Normal These findings are most consistent with: A. Hemophilia A B. Factor X deficiency C. Factor XI deficiency D. Von Willebrand's disease

Von Willebrand's disease

Which group of molds characteristically produces large, ribbon-like hyphae that are irregular in diameter and contain only occasional septa? A. Dermatophytes B. Zygomycetes C. Dimorphic fungi D. Dematiaceous fungi

Zygomycetes

Which NPN is in the highest concentration in the blood? urea Creatinine

urea

Enterococci can be described as: A. able to grow in 6.5% NaCl B. usually alpha hemolytic C. a major cause of pharyngitis D. often carrying an antiphagocytic capsule E. All of these are true.

able to grow in 6.5% NaCl

Which of the following has rhizoids between the sporangiophores? A. Mucor B. Absidia C. Rhizopus D. Trichophyton

absidia

concentration of unknown equals

absorption unknown/absorption standard x concentration of standard

The extent to which measurements agree with the true value of the quantity being measured is known as the:

accuracy

Which of the following makes a patient more prone to hemorrhage? A. factor V Leiden B. protein C deficiency C. free protein S deficiency D. acute disseminated intravascular coagulation (DIC)

acute disseminated intravascular coagulation (DIC)

Chronic myeloid leukemia (CML) often progresses to: A. chronic neutrophilic leukemia B. chronic idiopathic myelofibrosis C. acute leukemia, either myeloid or lymphoid D. chronic lymphocytic leukemia

acute leukemia, either myeloid or lymphoid

Which one of the following is the most common form of childhood leukemia? A. acute lymphocytic leukemia B. acute granulocytic leukemia C. acute monocytic leukemia D. chronic granulocytic leukemia

acute lymphocytic leukemia

An adult patient has a white blood count of 80 x 109/L. The differential has 92% Sudan Black B-positive blasts and 8% lymphocytes. Which of the following is most likely? A. acute myeloid leukemia, minimally differentiated B. acute myeloid leukemia without maturation C. immature B cell acute lymphoid leukemia D. acute megakaryocytic leukemia

acute myeloid leukemia without maturation

Increased total LD activity, confined to fractions 4 and 5, is most likely to be associated with: acute viral hepatitis myocardial infarction

acute viral hepatitis

The principle energy source for mature red blood cells is: A. Krebs cycle B. hexose monophosphate pathway C. adenosine triphosphate (ATP) generated from anaerobic glycolysis D. adenosine diphosphate (ADP) generated from aerobic glycolysis

adenosine triphosphate (ATP) generated from anaerobic glycolysis

Which of the following proteins has the FASTEST electrophoretic mobility?

albumin

A critically ill patient becomes comatose. The physician believes the coma is due to hepatic failure. The most helpful assay in this diagnosis would be: GGT ammonia

ammonia

Which of the following will cause spuriously prolonged clot-based coagulation tests? A. an evacuated 3.2% sodium citrate tube filled to 90% capacity B. an evacuated 3.2% sodium citrate tube filled to 50% capacity C. prolonged tourniquet application D. plasma collected from a traumatic tap

an evacuated 3.2% sodium citrate tube filled to 50% capacity

On what does factor VIII depend for stability? A. Platelets B. von Willebrand Factor C. protein S D. thrombin

von Willebrand Factor

Toxic granulation, Döhle bodies, and vacuolization in neutrophils are often found together in: A. May-Hegglin anomaly B. bacterial infection C. Chédiak-Higashi syndrome D. Alder-Reilly anomaly

bacterial infection

Which of the following is not a component of a basic single beam spectrophotometer:

beam splitter

Lymphocytes become transformed when they are: A. being stimulated by an antigen B. undergoing mitosis C. dying D. moving to secondary lymphoid tissues

being stimulated by an antigen

Which of the following is normal in a patient regularly taking aspirin? A. platelet count B. bleeding time C. synthesis of thromboxane A2 D. platelet cyclooxygenase function

bleeding time

Which of the following is normal in a patient regularly taking aspirin? A. platelet count B. bleeding time C. synthesis of thromboxane A2 D. platelet cyclooxygenase function

bleeding time

Which of the following is part of primary hemostasis? A. Fibrinolysis B. blood vessels C. Coagulation factors D. Inhibitors

blood vessels

Which of the following is part of primary hemostasis? A. Fibrinolysis B. blood vessels C. Coagulation factors D. Inhibitors

blood vessels

Chloramphenicol was an important antimicrobial agent for the treatment of pediatric meningitis as well as several other significant infections. Unfortunately, chloramphenicol exhibits a major complication that limits its clinical use. These effects include: A. allergic reactions B. bone marrow suppression and aplastic anemia C. significant gastrointestinal manifestations D. photosensitivity

bone marrow suppression and aplastic anemia

In the condition Kernicterus, the abnormal accumulation of bilirubin occurs in what tissue?

brain

Serum alkaline phosphatase activity is derived from all of the following organs except:

brain

Kernicterus is an abnormal accumulation of bilirubin in:

brain tissue

On what does factor VIII depend for stability? A. Platelets B. von Willebrand Factor C. protein S D. thrombin

von Willebrand Factor

A hospitalized patient is experiencing increased neuromuscular irritability (tetany). Which of the following tests should be ordered immediately?

calcium

The primary clinical usefulness of the -fetoprotein (AFP) assay as a tumor marker is:

checking tumor recurrence in patients with hepatocellular carcinoma

A middle-aged man has a white blood cell (WBC) count of 80 x 109/L with 90% lymphocytes and many smudge cells. Which of the following is most likely? A. chronic lymphocytic leukemia (CLL) B. hairy cell leukemia C. T cell lymphoma D. Sézary syndrome

chronic lymphocytic leukemia (CLL)

The function of 10% potassium hydroxide (KOH) in the direct examination of skin, hair, and nail scrapings is to: A. preserve fungal elements B. kill contaminating bacteria C. clear and dissolve debris D. fix preparation for subsequent staining

clear and dissolve debris

What are the primary procoagulant roles of the blood vessel? A. secrete platelet growth factors and manufacture calcium B. prevent the blood from escaping and promote fibrinolysis C. constrict and provide collagen and other compounds to activate clotting D. constrict and synthesize most plasma coagulation factors

constrict and provide collagen and other compounds to activate clotting

Iontophoresis is a technique used in the diagnosis of: pernicious anemia cystic fibrosis

cystic fibrosis

Pancytopenia is a term that means: A. increase in red and white cells B. decrease in red cells, white cells, and platelets C. decrease in any two blood cell lines D. increase in white cells and platelets

decrease in red cells, white cells, and platelets

A patient has an increased red count, hemoglobin, and hematocrit. Which of the following features points to secondary polycythemia over polycythemia vera (PV)? A. increased white count B. increased red cell mass C. bone marrow erythroid hyperplasia D. decreased arterial O2 saturation

decreased arterial O2 saturation

A cardiac glycoside that is used in the treatment of congenital heart failure and arrhythmias by increasing the force and velocity of myocardial contraction is

digoxin

A patient's blood smear shows a moderate number of spherocytes and polychromasia. Select the best test to distinguish warm autoimmune hemolytic anemia (WAHA) from hereditary spherocytosis. A. reticulocyte count B. direct antiglobulin test (DAT) C. bilirubin D. haptoglobin

direct antiglobulin test (DAT)

Which of the following is normal in hereditary spherocytosis (HS)? A. red cell morphology B. reticulocyte count C. direct antiglobulin test (DAT) D. osmotic fragility

direct antiglobulin test (DAT)

Which of the following would you find in a primary hyperparathyroidism case? decreased serum calcium and increased serum phosphorus elevated serum calcium and decreased serum phosphorus

elevated serum calcium and decreased serum phosphorus

Oral anticoagulation is based on: A. vitamin K antagonism B. combining with antithrombin C. increasing plasminogen D. activating plasmin

vitamin K antagonism

Which of the following would you find in a primary hyperparathyroidism case? elevated serum calcium and decreased serum phosphorus elevated serum calcium and elevated serum phosphorus decreased serum calcium and decreased serum phosphorus decreased serum calcium and increased serum phosphorus

elevated serum calcium and decreased serum phosphorus

In first-order kinestic the reaction is proportional to the amount of substrate because: enzyme and substrate are present in equal amounts enzyme amount is greater than substrate amount

enzyme amount is greater than substrate amount

Which hormone is produced by the kidney and influences erythrocyte production? A. Growth hormone B. erythropoietin C. hemoglobin synthetase D. ALA synthetase

erythropoietin

A patient has a platelet count of 1200 x 109/L. Many platelets are giant and have abnormal shapes. A mild anemia is present. The bone marrow has increased megakaryocytes in clusters; iron stores are present. Which of the following is most likely? A. chronic myeloid leukemia B. essential thrombocythemia C. polycythemia vera (PV) D. thrombocytosis secondary to blood loss

essential thrombocythemia

What important function does 2,3-biphosphoglycerate (2,3-BPG) perform? A. maintains iron in the hemoglobin molecule in the ferrous state B. prevents oxidative injury to the red cell C. facilitates the delivery of oxygen to tissue D. aids in the exchange of membrane lipids with lipids in plasma

facilitates the delivery of oxygen to tissue

What factor is fibrinogen? factor II factor X factor IV factor I

factor I

What factor is prothrombin? factor II factor X factor IV factor I

factor II

what is the prothrombin factor? A. factor III B. factor IV C. factor II D. factor X

factor II

what is the labile factor or proaccelerin factor? A. factor IV B. factor VIII C. factor II D. factor VI

factor IV

what is the labile factor or proaccelerin factor? A. factor IV B. factor VIII C. factor II D. factor V

factor V

what is the stable factor or proconvertin factor? A. factor I B. factor VII C. factor XII D. factor III

factor VII

What factor is Von Willebrand factor? factor VII factor X factor VIII factor III

factor VIII

The bleeding time result will be normal and thus of no value to perform in patients who have: A. von Willebrand disease B. factor VIII deficiency C. a qualitative platelet disorder D. Glanzmann thrombasthenia

factor VIII deficiency

what is the plasma thromboplastin antecedent? A. factor III B. factor VI C. factor XI D. factor V

factor XI

what is the Hageman factor? A. factor III B. factor X C. factor XII D. factor V

factor XII

what is the Hageman factor? A. factor III B. factor X C. factor XII D. factor VI

factor XII

what is the fibrin-stabilizing factor? A. factor VII B. factor IV C. factor XII D. factor XIII

factor XIII

Select the next best test to run given the following partial thromboplastin time (PTT) results on a patient's plasma: Initial PTT: 75 seconds Immediate 1:1 mix with normal plasma: 38 seconds Incubated 1:1 mix with normal plasma: 36 seconds A. dilute Russell's viper venom time (DRVVT) B. factor V Leiden C. Bethesda titer D. factor assays

factor assays

Macrophages are actually lymphocytes that have entered the tissues and body fluids via diapedesis. A. True B. False

false

Which of the following will give the best overall picture of a patient's iron stores:

ferritin

Which of the following deficiencies is associated with neural tube defects?

folate

A patient has 30% hemoglobin F. Which of the following can be eliminated? A. four missing genes for alpha chain production B. heterozygous hereditary persistence of fetal hemoglobin C. thalassemia intermedia D. the patient is a normal 1-month-old infant

four missing genes for alpha chain production

Zollinger-Ellison syndrome is characterized by a large (usually 20-fold) elevation of: glucagon gastrin

gastrin

Two children in the same family have recurrent, chronic bruising in multiple sites, repeated long-lasting epistaxis and minor bleeding after dental work. Which type of disorder is most likely? A. localized, congenital, anatomic B. generalized, congenital, systemic C. localized, acquired, anatomic D. generalized, acquired, systemic

generalized, congenital, systemic

What do all chronic myeloproliferative disorders (MPDs) share? A. Philadelphia chromosome B. increased red cell mass C. increased blood cells; overlapping clinical and laboratory features D. serious thromboembolic complications

increased blood cells; overlapping clinical and laboratory features

Oral anticoagulation is based on: A. vitamin K antagonism B. combining with antithrombin C. increasing plasminogen D. activating plasmin

vitamin K antagonism

A patient is stabilized on warfarin therapy and being monitored using the prothrombin time (PT) followed by calculation of the international normalized ratio (INR). The formula for calculating the INR is: INR = (PTpatient/PTnormal)ISI where ISI = international sensitivity index. What is used for the PTnormal? A. geometric mean prothrombin time reported by the manufacturer of the PT reagent being used B. geometric mean prothrombin time for the reference population C. arithmetic mean prothrombin time as found in a standard reference textbook D. arithmetic mean prothrombin time for the control

geometric mean prothrombin time for the reference population

The conversion of glucose or other hexoses into lactate or pyruvate is called: glycolysis glycogenolysis

glycolysis

A patient has an increased uric acid and a normal BUN and creatinine. What is the patient's most likely diagnosis?

gout

Organisms belonging to the genus Neisseria are: A. gram-positive diplococci B. gram-negative diplococci C. gram-negative coccobacilli D. gram-negative bacilli

gram-negative diplococci

The period of time during which the concentration of a therapeutic drug decays by 50% is referred to as the:

half-life of a drug.

An increased reticulocyte count MAY be found in all of the following conditions EXCEPT: A. hemolytic anemias B. following acute hemorrhage C. inflammatory response D. satisfactory response to therapy for pernicious anemia

hemolytic anemias

A 4-year-old child becomes very sick after eating undercooked ground beef. Kidney function tests are very elevated. Thrombocytopenia is mild and there are a few schistocytes on the blood smear. Which of the following is most likely diagnosis? A. immune thrombocytopenic purpura (ITP) B. hemolytic uremic syndrome (HUS) C. thrombotic thrombocytopenic purpura (TTP) D. disseminated intravascular coagulation (DIC)

hemolytic uremic syndrome (HUS)

A 4-year-old child becomes very sick after eating undercooked ground beef. Kidney function tests are very elevated. Thrombocytopenia is mild and there are a few schistocytes on the blood smear. Which of the following is most likely diagnosis? A. immune thrombocytopenic purpura (ITP) B. hemolytic uremic syndrome (HUS) C. thrombotic thrombocytopenic purpura (TTP) D. disseminated intravascular coagulation (DIC)

hemolytic uremic syndrome (HUS)

Renal failure is a prominent feature of: A. hemolytic uremic syndrome (HUS) B. thrombotic thrombocytopenic purpura (TTP) C. Plasmodium malariae infection D. march hemoglobinuria

hemolytic uremic syndrome (HUS)

Specimens for blood gas determinations should be drawn into a syringe containing: no preservative heparin

heparin

The only acceptable anticoagulant for a calcium determination is:

heparin

Moderate to marked target cells are found on a blood smear. Which of the following can most likely be eliminated? A. hemoglobin C disease B. hereditary Spherocytosis C. hemoglobin E disease D. liver disease

hereditary Spherocytosis

Cells with a slit (stomatocytes) instead of a round central pallor can be found in all of the following except: A. hereditary spherocytosis (HS) B. smears with a drying artifact C. Rh null disease D. liver disease

hereditary spherocytosis (HS)

The granules that basophils contain are composed of: A. histamine B. serotonin C. peroxidase D. a "thromboplastin-like" substance

histamine

A patient has the following test results: Increased serum calcium levels, Decreased serum phosphorus levels, Increased levels of parathyroid hormone. This patient most likely has: hyperparathyroidism hypoparathyroidism hyperthyroidism hypothyroidism

hyperparathyroidism

Calcium and phosphorus levels were determined for a 4-year-old patient as follows: Ca - 14mg/dl Phosphorus - 1mg/dl These results are MOST compatible with: rickets hyperparathyroidism hyperthyroidism

hyperparathyroidism

Respiratory alkalosis is commonly caused by:

hyperventilation

A child has monotonous blasts in the peripheral blood that have scant blue cytoplasm. They are CALLa (CD10) positive. Which form of acute lymphoid leukemia (ALL) is most likely? A. immature B cell B. immature T cell C. mature B cell D. mature T cell

immature B cell

Homozygous alpha-thalassemia (--/--) has what clinical outcome? A. incompatible with life B. severe anemia C. no anemia D. mild anemia

incompatible with life

Patients who develop severe sepsis or septic shock commonly have __________ plasma lactic acid values.

increased

Laboratory professionals are at special risk for disease transmission. The majority of cases of laboratory-related infections are associated with: A. infectious aerosols B. contamination of abraded skin C. puncture wounds D. bite of a lab animal

infectious aerosols

In a specimen collected for plasma glucose analysis, sodium fluoride:

inhibits glycolysis

Depletion of serum haptoglobin indicates: A. decreased erythropoiesis B. extravascular hemolysis C. intravascular hemolysis D. increased phagocytosis of macrophages

intravascular hemolysis

The presence of schistocytes in the peripheral blood indicates which of the following processes? A. iron deficiency anemia B. intravascular hemolysis C. megaloblastic anemia D. extravascular hemolysis

intravascular hemolysis

Absorption of vitamin B12 in the stomach and gut requires the presence of: folic acid intrinsic factor

intrinsic factor

Transferrin receptors can be measured and are elevated when: A. erythropoietin increases B. transferrin increases C. iron stores are decreased D. chronic disease is present

iron stores are decreased

Non-insulin-dependent diabetes:

is associated with resistance to the action of insulin

An increase in the concentration of 2,3-biphosphoglycerate (2,3-BPG): A. increases the affinity of hemoglobin for oxygen B. is present in patients with severe anemia and decreased blood pH (acidosis) C. shifts the oxygen dissociation curve to the left D. puts hemoglobin the relaxed conformation form

is present in patients with severe anemia and decreased blood pH (acidosis)

Secretion of renin and antidiuretic hormone (ADH) is induced by low blood pressure and volume. Renin is synthesized in the _____ and ADH is made in the _____.

kidney, brain

Phenylketonuria is caused by: excessive ingestion of milk products containing phenylalanine. inability to metabolize tyrosine. lack of the enzyme phenylalanine hydroxylase. a mousy odor in the urine.

lack of the enzyme phenylalanine hydroxylase.

The wet mount and Gram stain of vaginal secretions from a patient who has bacterial vaginosis caused by Gardnerella vaginalis will reveal: A. large, squamous epithelial cells with numerous attached pleomorphic gram-variable or gram-negative coccobacilli and rods. B. large numbers of large gram-positive rods. C. large numbers of polymorphonuclear leukocytes. D. All of these are correct.

large, squamous epithelial cells with numerous attached pleomorphic gram-variable or gram-negative coccobacilli and rods.

Plasma proteins are primarily synthesized in the:

liver

What organ is associated with the production of the majority of clotting factors? A. Thymus B. Duodenum C. Liver D. Kidney

liver

Which of the following results in ineffective thrombopoiesis? A. megaloblastic anemia B. thrombotic thrombocytopenic purpura (TTP) C. disseminated intravascular coagulation (DIC) D. immune thrombocytopenic purpura (ITP)

megaloblastic anemia

Ringed sideroblasts are a reflection of iron-laden: A. Golgi complex B. rough endoplasmic reticulum C. mitochondria D. ribosomes

mitochondria

Clinical assays for tumor markers are MOST important for:

monitoring the course of a known cancer

Malignant proliferation of plasma cells with osteolytic bone lesions and the presence of monoclonal immunoglobulin in the serum describe: A. Waldenström Macroglobulinemia B. marginal zone B cell lymphoma C. follicle center lymphoma D. multiple myeloma

multiple myeloma

The term antigenic drift refers to: A. mutation during genome replication resulting in minor antigenic change and relatively mild influenza outbreaks every 1 to 3 years. B. reassortment of the segmented viral genome during co-infection in nonhuman animals, resulting in major antigenic change and periodic worldwide outbreaks. C. what happens when human virus infects a cell at the same time as an animal influenza virus. D. All of these are correct.

mutation during genome replication resulting in minor antigenic change and relatively mild influenza outbreaks every 1 to 3 years

What is the earliest identifiable cell in the granulocyte maturation sequence? A. pluripotent stem cell B. myelocyte C. myeloblast D. CFU-GEMM

myeloblast

What is the earliest identifiable cell in the granulocyte maturation sequence? A. pluripotent stem cell B. myelocyte C. myeloblast D. CFU-GEMM

myeloblast

Which of the following is the EARLIEST biochemical marker of myocardial infarction? CK myoglobin

myoglobin

A 10-year-old female presents with the complaint of bone pain. ALP 3.5 higher than upper reference limit. These results are consistent with a diagnosis of: bone cancer normal bone growth

normal bone growth

During an evaluation of adrenal function, a patient had plasma cortisol determinations in the morning after awakening and in the evening. Laboratory results indicated that the morning value was higher than the evening concentration. This is indicative of: normal finding Cushing syndrome

normal finding

A patient has a white count of 15.0 x 109/L. What is the correct interpretation? A. leukocytosis for any patient B. normal for an infant, leukemoid reaction for an adult C. normal for an infant, leukocytosis for an adult D. normal for any patient

normal for an infant, leukocytosis for an adult

Which of the following is true of megakaryocytes as they mature? A. nucleus becomes polyploidy B. cytoplasmic basophilia becomes more pronounced C. size decreases D. nucleoli become more prominent

nucleus becomes polyploidy

Once a patient is stabilized on warfarin therapy, how often should she or he be monitored? A. once a month B. once every 6 months C. once a year D. once every week

once a month

Once a patient is stabilized on warfarin therapy, how often should she or he be monitored? A. once a month B. once every 6 months C. once a year D. once every week

once a month

Which of the following has a pyknotic nucleus? A. Pronormoblast B. basophilic normoblast C. polychromatophilic normoblast D. orthochromic normoblast

orthochromic normoblast

What tests should be performed on a regular basis on patients receiving therapeutic heparin? A. factor assays and platelet count B. partial thromboplastin time (PTT) and white count C. partial thromboplastin time (PTT) and platelet count D. prothrombin time (PT) and hematocrit

partial thromboplastin time (PTT) and platelet count

TSH is produced by the: pituitary gland hypothalamus

pituitary gland

In respiratory acidosis, a compensatory mechanism is the increase in: plasma bicarbonate concentration respiration rate

plasma bicarbonate concentration

What substance digests the fibrin clot after healing? A. alpha-2-antiplasmin B. plasmin C. tissue plasminogen activator (TPA) D. activated protein C

plasmin

Pernicious anemia (PA) can be distinguished from folate deficiency by the: A. presence of hypersegmented neutrophils B. mean cell volume (MCV) C. bone marrow findings D. presence of autoantibodies to intrinsic factor (IF)

presence of autoantibodies to intrinsic factor (IF)

Increased TSH, decreased T3 and decreased T4 with presence of goiter indicate: secondary hypothyroidism. primary hypothyroidism

primary hypothyroidism

Which of the following aids in preventing excess clotting? A. Thrombin B. factor VII C. calcium D. protein C

protein C

What tests should be performed on a regular basis on patients receiving therapeutic heparin? A. factor assays and platelet count B. partial thromboplastin time (PTT) and white count C. partial thromboplastin time (PTT) and platelet count D. prothrombin time (PT) and hematocrit

prothrombin time (PT) and hematocrit

The principle excretory form of nitrogen is:

urea

The Philadelphia chromosome: A. results in a chimeric gene (bcr/abl) B. is diagnostic for polycythemia vera (PV) C. is a balanced reciprocal translocation between chromosomes 7 and 19 D. manifests as a tyrosine kinase that blocks cell maturation

results in a chimeric gene (bcr/abl)

What condition is most associated with autosplenectomy? A. abdominal trauma B. thrombocytopenia C. sickle cell anemia D. iron deficiency

sickle cell anemia

Campylobacter spp. can sometimes be detected by direct Gram stain examination of fecal sample, which would reveal many: A. large, faintly staining gram-negative bacilli B. small, curved or seagull-winged gram-negative bacilli C. small gram-negative coccobacilli D. plump, faintly staining gram negative bacilli

small, curved or seagull-winged gram-negative bacilli

Which electrolyte level best correlates with plasma osmolality?

sodium

Which of the following electrolytes is the chief plasma cation whose main function is maintaining osmotic pressure?

sodium

A culture performed on a vaginal specimen collected from a 60-year-old woman would likely grow primarily: A. group B beta-hemolytic streptococci. B. staphylococci and corynebacteria. C. Enterobacteriaceae, streptococci, staphylococci, lactobacilli, and anaerobes. D. Gardnerella vaginalis, Prevotella spp., peptostreptococci, and Mobiluncus spp.

staphylococci and corynebacteria.

The qualitative platelet disorders and vascular disorders typically exhibit what type of symptoms? A. superficial bleeding B. hemarthrosis C. deep tissue bleeding D. thrombosis

superficial bleeding

Violation of the 2-2S, 4-1S or 10x Westgard rules is a strong indication of _________. random error trend systematic error shift

systematic error

A patient has a quantitative D-dimer of 800 ng/mL. Which of the following can be ruled out? A. Deep venous thrombosis (DVT) B. pulmonary embolus C. disseminated intravascular coagulation (DIC) D. the patient is normal

the patient is normal

The therapeutic range of a drug is:

the range of concentrations within which the drug is effective yet not toxic

Infection of the buccal (cheek) mucosa, tongue, or oropharynx by Candida spp. is called: A. pharyngitis. B. Vincent's angina. C. rhinitis. D. thrush.

thrush

An example of a hormone synthesized in the hypothalamus would be:

thyrotropin releasing hormone

Regarding thyroid hormones, the primary secretory product of the normal thyroid gland is in the form of

thyroxine (T4).

True or false? Chloride is the major extracellular anion in plasma.

true

A 16 year old female was taken to the emergency room in a coma by her mother. She had lost 25 pounds in 2 months; she was always thirsty; drinking a lot of water and running to the bathroom. Her laboratory results are shown below. Results: (indicates reference range) pH: 7.21 (7.35-7.45) pCO2: 23 mm Hg (35-45 mm Hg) Glucose: 742 mg/dL (60-110 mg/dL) Urine ketone: positive Urine glucose: large What is the MOST likely cause of this patient's symptoms?

type I diabetes mellitus

Which of the following is necessary for platelet adhesion to collagen? A. factor XIII B. fibrinogen C. calcium D. von Willebrand factor (VWF)

von Willebrand factor (VWF)

Which of the following is necessary for platelet adhesion to collagen? A. factor XIII B. fibrinogen C. calcium D. von Willebrand factor (VWF)

von Willebrand factor (VWF)

The protein "glue" that optimally binds platelets and collagen after vessel injury is: A. Fibronectin B. von Willebrand's Factor (VIII:vWF) C. Factor II D. Factor I

von Willebrand's Factor (VIII:vWF)

This trace element plays a key role in protein and nucleic acid synthesis: zinc copper

zinc

One type of sexual reproduction produces rough-walled spores called ______________, which are formed by the union of two matching types of Zygomycetes. A. sporangiophores B. zygospores C. zygoconidiaphores D. conidiophores

zygospores


Related study sets

Financial Markets and instituitions

View Set

Lesson 2 - Introduction to Forms of Energy

View Set

Basic Livestock Nutrition - Assessment V

View Set

AP Biology Chapter 54 Community Ecology

View Set

Ch 7. - Positive Organizational Behavior

View Set